Download as pdf or txt
Download as pdf or txt
You are on page 1of 144

1. Heat fixation should NOT be used for which of the following stains?

a. Capsular staining
b. Gram stain
c. Endospore staining
d. Acid-fast stain

2. Which of the following is NOT a primary purpose of fixation?

a. To prevent putrefaction
b. To prevent autolysis
c. To enhance differences in the refractive indexes of various tissue structures
d. To expose antigen sites for immunohistochemical staining

3. When processing delicate specimens using a standard closed tissue processor,


dehydration should be done by which of the following methods to minimize tissue
distortion?

a. A graded series of reagents of increasing concentration


b. A graded series of reagents of decreasing concentration
c. A single reagent at a single concentration
d. Delicate specimens do not require a dehydration step

4. Which of the following is an artifact of over-decalcification of bone tissue?

a. The slide appears to be covered in dust


b. Hematoxylin and eosin (H&E) stain shows poor nuclear (basophilic) staining
c. Large holes are present that could be mistaken for vacuoles
d. The tissue shows a "parched earth" cracking appearance

5. When orienting a tissue for embedding, which of the following tissues requires special
attention to ensure it is cut in cross section?

a. Brain
b. Liver
c. Fallopian tubes
d. Muscle biopsies

Answers & Explanations


1. A: Most bacteria produce a capsule, or glycocalyx, just outside the cell wall. This capsule is
usually made up of polysaccharides. Heat fixation will cause this moist slime layer to shrink,
making it difficult to see once stained. Also, heat fixing may cause a bacterial cell to shrink,
creating a clear zone around the cell that appears like a capsule when one does not truly exist.
Therefore, when staining to view a bacterial capsule, a sample is air-dried and then a negative
stain is generally used for visualization.
2. D: Fixation has many purposes including preventing autolysis and putrefaction, enhancing
differences in refractive indexes of tissue structures, maintaining proper relationship between
cells and extracellular substances, and making the tissue firmer so dissection and cutting is
easier. However, fixation can have the downside of masking antigenic sites, resulting in poor
immunohistochemical staining.

3. A: Dehydration should be done slowly. If the concentration gradient differs significantly


between the inside and the outside of the tissue, the resulting diffusion currents could produce
cell distortions. This is why slowly replacing the water through a graded series of reagents of
increasing concentration is necessary to maintain proper structure before clearing and subsequent
infiltration with a medium such as paraffin.

4. B: The most common problems associated with bone processing are bone dust, under-
decalcification, and over-decalcification. When dust created by the saw when sectioning is
pressed into the surface of the bone, the resulting slide appears to be covered in dust. Using a
saw with a diamond blade can prevent this problem. Under-decalcification makes section cutting
very difficult, resulting in fragmentation problems. Over-decalcification results in poor nuclear
staining.

5. C: While most tissues are embedded flat, some tissues require special orientation. Tubular
structures, such as fallopian tubes, should be embedded in cross section so that the lumen and all
layers can be seen. Tissues with an epithelial surface, such as skin, are oriented so that they are
cut in a plane at a right angle to the surface.

1. Which of the following reagents can be used both as a fixative and a staining agent?
A. Eosin.
B. Uranyl acetate.
C. Acid fuchsin.
D. Alizarin red.
E. Picric acid
Picric acid is used as a fixative in Bouin's fixative and as a staining agent in van Gieson stain.
The other chemical that can be used both as fixative and staining agent is osmium tetraoxide.
2. Which of the following stain(s) involve(s) an oxidation step at the beginning of the
procedure?
1. Gomori's methenamine silver (GMS) netdod.
2. Von Kossa's silver deposition method.
3. Periodic acid Schiff (PAS) reaction.
4. Van Gieson stain.

A. 1, 2, and 3 are true.


B. 1 and 3 are true.
C. 2 and 4 are true.
D. Only 4 is true.
E. All of the above.
1. Gomori's methanamine silver (GMS).
3. Periodic acid Schiff (PAS).

Discussion: PAS and GMS stain are often used to detect fungal organisms such as Histoplasma, Candida,
Blastomyces and other fungal organisms. GMS stain shares a similarity with PAS stain in asking for an
oxidizing step at the beginning of the procedure. In PAS stain, periodic acid is used and in GMS stain,
chromic acid is used. At this initial step, the hydroxyl groups (-OH) will be oxidized to aldehyde groups (-
CHO). In GMS stain, the tissue is then treated with methenamine silver solution. Silver ions will be
reduced to silver metal by the aldehyde groupsand give a black color. In PAS stain, the aldehyde group
will react with the Schiff reagent to form an amide that gives the purple red color. In contrast to PAS
stain, chromic acid, however, is a strong oxidizing agent and will oxidize some of the aldehyde group
further to substance that would not be able to react with silver ion. With this token, the background
produced by collagen and basement will be suppressed. Light green is used as a counterstain in GMS
stain.

Von Kossa's silver deposition method is used to detect calcium. It involves the exposure of slides in
silver ion containing solution to strong light but no oxidizing agents are involved. Van Gieson stain is
used to study connective tissue and does not involve any oxidizing steps.

3. Sodium thiosulfate (or "Hypo") is used in the type of stain that is illustrated here. The
function of sodium thiosulfate is:

A. To activate the molecules to react with silver ion.


B. To harden the tissue.
C. To differentiate the intensity of the staining.
D. To remove unreacted silver ions.
E. To tone down the staining.
Discussion: This photo is obtained from a brain with Alzheimer's disease. The senile plaques
(white arrow) and neurofibrillary tangles (black arrow) are well demonstrated. The color is
compatible with a modified Bielschowsky stain which is widely used in the study of
neurodegenerative diseases. Similar to many other silver stains, silver ions are present in the
staining solution and are reduced to silver metal that would deposit on the tissue section. The
unreacted silver ions are removed by sodium thiosulfate ("hypo"). The removal of unreacted
silver ions represents perhaps the most important discovery in the invention of emulsion based
photography as this step allows preservation of the images without getting dark with time due to
the slow reduction of silver ions to silver metal. This process is known as fixation in emulsion
based photography. Background staining and the intensity of the staining in many silver stains
can be toned down by using a solution containing gold, often gold chloride.
4. Which of the following stains uses osmium tetraoxide to stain for myelin?
A. Klüver-Barrera stain.
B. Marchi's impregnation method.
C. Weil's method.
D. Gallocyanin stain.
E. Pal-Weigert method.
Marchi's impregnation is based on selective blockage of osmium staining of normal
myelin by treatment with an oxidizing agent. The Swank-Davenport modification uses
potassium chorate as the oxidizing agent and appears to be perhaps the most reliable
method. Marchi's impregnation asks for en bloc impregnation with osmium tetraoxide
before sectioning. Degenerating myelin will stain black.

All others but Gallocyanun stain are myelin stains. Klüver-Barrera stain is a combined
stain of Luxol fast blue for myelin and cresyl violet for neurons. Weil's method and Pal-
Weigert method are also myelin stains. All these stains would give myelin a blue to
dark-blue stain. Gallocyanin-alum method can be used to demonstrate nucleic acid and
will give the Nissl substance a blue color. Non of these stains contain osmium.
5. Which of the following is not true about terminal deoxynucleotidyl transferase
mediated dUTP nick end labeling (TUNEL)?
A. This process labels the nick ends of DNA.
B. This process is sequence dependent.
C. This process can be performed on a section or in a liquid sample.
D. This process can be used to detect apoptotic cells on sections in many
situations.
E. This process can be used with enzyme linked method or fluorescent dye in
order to be visualized on tissue sections.
Discussion: Everything else being mentioned are true for TUNEL except that it is not a
sequence dependent. It simply adds nucleotides to the nicked end of DNA.
6. Sections to be viewed under the electron microscope (ultra thin sections) are usually
cut at what thickness?
A. 5-10 nm.
B. 60-90 nm.
C. 190-240 nm.
D. 0.5-1.0 m.
E. None of the above.
Discussion: For diagnostic transmission electron microscopy, the resin embedded sections
are cut at 60-90 nm. The so called semi-thin resin embedded sections for light microscopy,
however, are cut at 0.5-1.0 m. Semi-thin sections are most often used for identification and
targeting of tissue as well as pathologic examination of peripheral nerve biopsy.
7. What is the artifact being illustrated in this hematoxylin-eosin stained intraoperative
cytologic preparation from a pituitary adenoma?
A. Left in methanol for too long.
B. Left in air for too long before fixation with methanol.
C. Left in xylene for too long.
D. The hematoxylin is not functioning properly.
E. There is no artifact here.

Diagnosis: The artifacts being shown here are air dry artifacts because the unstained
smear has been left in air for too long before fixation during intraoperative consultation.
Thes artifacts can be seen in smear as illustrated here or in frozen sections. This tissue
in this question was obtained from a pituitary adenoma. After the smear has been
made, it should be fixed immediately. The author of this question prefers to use ethanol
as fixative and have a container of methanol that would be reached within one second
after the smear has been made. You can tell the difference between preparations with
and without air dry artifacts. The cytologic details are all lost in the smear with air dry
artifact. Both slides are obtained from the same pituitary adenoma and the photos are
taken at the same magnification. Please note that nuclei with air dry artifacts are
markedly blown up in size. One must be cautious when air dry artifact is not as dramatic
as the one being shown here as the air dry artifact would blow up the size of the nuclei
and lead to a wrong impression of increased nuclear size and pleomorphism during
intraoperative consultations.

Leaving slides in alcohol and xylene for a prolonged period of time usually do not
produce any noticeable artifacts. No hematoxylin can give you proper nuclear details
and staining if there is air dry artifact.
8. Which of the following(s) is(are) true in the illustration here regarding the stain being
used for this cytologic preparation?
1. The stain is most consistent with a DiffQuick stain.
2. The slide should be thoroughly air dried before staining.
3. This type of stain is commonly used for the examination of cytologic specimens
in North America.
4. Similar results can be obtained in smears with other Romanowsky family of
stains.

A. 1, 2, and 3 are true.


B. 1 and 3 are true.
C. 2 and 4 are true.
D. Only 4 is true.
E. All of the above.
9. The numerical aperture of the optical pathway of a microscope is:
1. Limited by the numerical aperture of the objective.
2. Affected by the condenser diaphragm.
3. Limited by the the refractive index of the medium of the optical pathway.
4. Determined and limited by the type of illumination (i.e., whether it is bright field,
dark field or fluorescent).

A. 1, 2, and 3 are true.


B. 1 and 3 are true.
C. 2 and 4 are true.
D. Only 4 is true.
E. All of the above.
The resolution of an optical microscope is limited by the numerical aperture. The higher the
numerical aperture the higher the resolution. The numerical aperture of the optical system is
limited by the numerical aperture of the objective and also by the numerical aperture of the
condenser. The numerical aperture can be further reduced by closing down the diaphragm of
the condenser. The numerical aperture is also limited by the refractive index of the medium of
the optical pathway between the condenser and the slide, and between the slide and the
objective. Air has a refractive that is very close to 1.00 and therefore no dry objectives can
exceed this numerical aperture. On the other hand, numerical aperture of oil or other fluid
immersion objectives can exceed 1.00 because the refractive index of the oil and the fluid can
exceed 1.00. It is important to note that oil or the immersion fluid has to be applied in between
the glass slide and the objective and also the glass slide and the condenser to achieve the
maximum numerical aperture and therefore the highest resolution. The light source does not
affect the numerical aperture.
10. Which of the following reagent(s) is(are) used in electron microscopy?
A. Uranyl acetate.
B. Osmium tetraoxide.
C. Lead nitrate.
D. All of the above.
E. None of the above.

Discussion: The reason for using osmium, lead, and uranium to stain sections for
transmission electron microscopy is because these heavy metals can absorb electrons.
11. Which of the following fixatives does not contain heavy metal?
A. Zenker's fixative.
B. Osmic acid fixatives.
C. B-5 fixative.
D. Bouin's fixative.
E. Olmacher's fixative.
Zenker's fixative, Olmacher's fixative, and B-5 solution contain mercury. Osmic acid fixative
contain osmium but this fixative is rarely used today for histopathology. It is still used for fixation
for electron microscopy for some purposes. Bouin's solution does not contain any heavy metal.
12. The paraffin sections for routine diagnostic surgical pathology are cut at a thickness
of:
A. 4-8 nm.
B. 40-80 nm.
C. 4-8 m.
D. 40-80 m.
E. None of the above.
Most paraffin sections for hematoxylin-eosin stains for routine diagnostic purposes are cut at 4-6
m, sometimes at 8 m. Some special stains may ask for thicker sections.
13. Which of the following(s) is(are) true regarding this preparation that is obtained from
a normal anterior pituitary gland?

1. The stain being used demonstrates the reticulin fibers.


2. The stain being used is a Verhoeff elastic stain.
3. The staining mechanism involves reducing activated silver ion into silver metal.
4. The staining solution being used is composed of a hematoxylin with high
concentration of ferric chloride.

A. 1, 2, and 3 are true.


B. 1 and 3 are true.
C. 2 and 4 are true.
D. Only 4 is true.
E. All of the above.
Discussion: This photo demonstrated the reticulin fibers in the anterior pituitary. Reticular
fibers can be demonstrated by Gordon and Sweet's method or Gomori's method. Both methods
involve the reduction of silver ion into silver metal and its deposition on the reticulin fiber would
turn them black. Verhoeff elastic stain involves hematoxylin with high concentration of ferric
chloride and stains elastic fibers black.
14. The thickness of ultra thin sections for electron microscopy can be estimated by?
A. The readings on the telescope that is attached to the ultra-microtome.
B. The time it takes for the section to sink to the bottom of the water trough.
C. The reflected color of the floating tissue section.
D. The darkness of the section through transmitted light.
E. The thickness can only be estimated with the electron microscope.
The thickness can be estimated by the reflected color of the tissue when it is floated on water.
Most sections for diagnostic purposes are cut at 60-90 nm and would give a silver reflection.
Any section under 60 nm will give a gray reflection. Sections that are thicker then 90 nm will
carry a gold, purple, and blue reflection as the thickness goes from 90 to 240 nm.
15. What is the chemical that is used to produce the brown color in this peroxidase
mediated immunohistochemistry preparation that is performed with a primary
antibody specific for synaptophysin?

A. Fast red TR.


B. 3-amino-9-ethylcarbazole.
C. Diaminobenzidine.
D. Nitro blue tetrazolium.
E. Hexazotized new fuchsin.

Discussion: Both diaminobenzidine (DAB) and 3-amino-9-ethylcarbazole. (AEC) are


substrates for peroxidases. While DAB gives a brown product, AEC gives a red product. Fast
red TR, nitro blue tetrazolium, and hexazotized new fuchsin are substrates for alkaline
phosphatase and they would give a red, dark-blue, and red product respectively. Nitro
blue tetrazolium develops rather slowly and allows close monitoring of the development.
This feature allows tight control of the staining process and is often used in in situ
hybridization.
16. Which of the following is not true about in situ hybridization?
A. It is used to detect a specific sequence of DNA or RNA in tissue section.
B. The process involves annealing of the probe and the target polynucleotide
sequence.
C. The signal can be generated by enzyme-linked, fluorescent, or radioactive
methods.
D. The probe must be DNA in nature.
E. There is a limitation on the length of the probe.
Discussion: In situ hybridization can be used to detect the presence or absence of a specific
sequence of DNA or RNA in tissue sections, cytologic preparations, and sometimes whole
mounts of tissue. It is sequence specific. The process involves annealing of the probe
polynucleotide with the target sequence. Probe sequences longer then 500 base pair will have
steric hindrance and will be difficult to hybridize. The probe, however, can be made of both DNA
or RNA (riboprobe). Signal can be visualized with enzyme-linked or fluorescent methods.
Radioactive signal can also be visualized by photographic emulsions (autoradiography).
17. Which of the following reactions is the best choice to demonstrate only DNA?
A. Gallocyanin-chrome alum method.
B. Feulgen reaction.
C. Hematoxylin stain.
D. Solochrome cyanine method.
E. Thioflavine S stain.
Discussion: Gallocyanin-chrome alum method will stain both DNA and RNA dark blue and,
therefore, cannot provide a good estimate on the amount of DNA in the cell. The same is true
for hematoxylin. Thioflavine S stain can be used to demonstrate amyloid deposition and must be
visualized with a fluorescent microscope. It cannot be used to estimate the amount of DNA in
tissue sections. Solochrome cyanine method can be used to differentiate osteoid from newly
laid-down bone and older bone but does not stain DNA specifically. Feulgen reaction stains
DNA red-purple and the cytoplasm green and is a widely accepted method to demonstrate DNA
on histologic sections.
18. This is a portion of the cerebral cortex and white matter stained by modified
Bielschowsky stain The findings being illustrated here are most consistent with:

A. "Swiss cheese" artifact due to gas production resulted from post mortem
bacterial overgrowth.
B. The specimen has been fixed in formalin and then frozen (freezing artifact).
C. Over digestion by enzyme at the initial steps as required by modified
Bielschowsky stain.
D. Wax cracks due to in proper processing of the tissue.
E. Knife marks.
Discussion: "Swiss cheese" artifacts can be generated by post mortem over growth of
gas producing bacteria. The size can vary from millimeters to over 1 centimeters. Just
like the vacuoles in Swiss cheese, their distribution is irregular and there is a significant
variation in size. In contrast to the vacuoles being shown here, the vacuoles in "Swiss
cheese" artifact have smooth margins. Although enzyme digestion can be used to
retrieve signal in immunohistochemistry and in situ hybridization, it is not a required step
for modified Bielschowsky stain which is a silver stain that is often used in the study of
neuropathology. In addition, the holes that are created by over digestion is never so
sharply defined. Wax cracks typically appear as irregular, elongated slits in the tissue
section. Knife marks are straight, very long and often appear as parallel lines. These
features are not present here.

In the section being shown, there is a randomly regular distribution of vacuoles of


comparable size in the grey matter. The extent of involvement is similar in different parts
of the photo. There is also similar involvement in the white matter albeit the the
vacuoles are far smaller. This difference is well appreciated in the medium
magnification photo. This type of artifact is produced by freezing tissue that has been
fixed in formalin. This brain has been fixed in formalin and solidly frozen before paraffin
sections are made. This type of artifact can be avoided by cryoprotection with
concentrated sucrose solution if frozen sections are to be done on formalin fixed tissue
blocks.
19. Which of the followings is not true for the succinate dehydrogenase preparation of
muscle biopsy as illustrated here?
A. This preparation detects an enzyme that is present in both the mitochondria
and tubules.
B. This preparation can be used to detect abnormal mitochondrial accumulation
in mitochondrial diseases.
C. The substrate will turn blue if the enzyme is present in the biopsy tissue.
D. Type I fibers will stain dark and type II fibers will stain pale.
E. This preparation can be used to demonstrate the core in central core disease.
Discussion: In contrast to NADH-TR, succinate dehydrogenase is present only in the
mitochondria
20. The light and dark bands that are perpendicular to the long axis of the muscle fiber in
this resin embedded, toulidine blue stained semi-thin (0.5-1 m) section are:

A. Knife marks.
B. Caused by inhomogeneous curing of the block.
C. Uneven staining ripples.
D. Caused by vibration of the block, the block holder, or the knife.
E. Insufficient osmium concentration leading to softening of the block.
Discussion: The ripple-like light and dark bands observe a rather strict periodicity and
are perpendicular to cutting action of the ultramicrotome. This type of artifact is typically
caused by a high frequency vibration of the cutting system that involves either the block,
the block holder, or the knife. The vibration introduces a forward and backward type of
movement of the block against the knife during sectioning. How to tell from this photo?
There is a knife mark here (delineated by the white arrows). Knife marks are always
perpendicular to the direction of movement of the block on sectioning.

Knife marks are never this periodical nor this wide as illustrated here. Insufficient
osmium or inappropriate curing will not lead to this type of artifact. Inhomogeneous
staining will lead to rather distinct patches of under stained or unstained areas with a
blur margin. On the other hand, there is a faint staining water mark due to suboptimal
staining as illustrated by the red arrow. Please note the curvy shape of this water mark
which reflects the rolling shape of the section because of the vibration artifact (red
arrow). Also note that the section above this curvy line is slightly more darkly stained
then the section below this line.
21. Hematoxylin stain with progressive formulae:
A. Are widely used by many first year residents who want to make progress and
be promoted to the second year.
B. Are used only by scientists who already had stellate achievements, such as
getting the Nobel prize.
C. Are used to remediate medical students who have made no progress during
their pathology rotation.
D. Are designed for the automatic staining machines so that slides racks can
progress forward.
E. Give a darker staining intensity with longer staining time.
Discussion: The first four answers are obviously jokes. I hope they would serve as the spices
of your day. In general, hematoxylin stains are divided into the so-called progressive formulae
and regressive formulae. The progressive formulae stain far slower then that of the regressive
formulae and therefore the staining intensity can be controlled by the length of the staining time.
Hematoxylin stains with regressive formulae involve over staining the sections followed by a
differentiation step to achieve the desired staining intensity. The regressive formulae are
particularly useful for tissues that do not take up hematoxylin well. One of the best examples is
over decalcified tissue.
1. Which of the following is a confirmatory test for syphilis:
a. RPR
b. FTA-ABS
c. VDRL
d. Wasserman test

Feedback: The FTA-ABS is not intended for use as a screening test. It should only be used to
differentiate between true positives, false positives, and in late or latent syphilis

2. The most likely cause of an elevated potassium level in an apparently normal individual is:
a. Contamination
b. Hemolysis
c. Acute renal failure
d. Interfering substances

Feedback: Hemolysis could result from improper specimen collection technique, or


improper storage.

3. Metabolic acidosis is characterized by:


a. Increased pCO2
b. Hypoventilation
c. Low Ph
d. High pH
Feedback: Metabolic acidosis is the result of either: 1) accumulation of abnormal levels of
organic acids,often secondary to diabetic ketoacidosis, or lactic acidosis, 2) excessive loss of
bicarbonate asin severe diarrhea, or 3) reduced excretion of acids as in renal failure. All these
conditionsresult in a decrease in pH. The body compensates by hyperventilating, and lowering
pCO2, inan attempt to restore normal pH, resulting in a compensated or partially
compensatedmetabolic acidosis
4. The Quelling test is useful for which of the following :
a. Differentiate between Staphylococcus andStreptococcus
b. Serological typing of Streptococcus pneumonia
c. Isolation of various strains of Staphylococcus
d. Isolation of various strains of Streptococcus

Feedback: Capsular swelling results from the addition of specific capsular antibody to an
isolate of the organism. This test is generally not required for routine diagnosis.

5. The relative centrifugal force of a centrifuge may be affected by all except the following:
a. Time
b. Radius of head
c. RPM
Feedback: The amount of time a sample is spun does not affect the relative centrifugal force
6. Standard precautions means that:
a. Known infectious patients must be handled with extraordinary care
b. Mask, gloves and gowns should be worn in all hospital patient's rooms
c. All specimens must be handled as if they are hazardous and infectious
d. Always wash your hands before drawing a patient’s blood sample
7. RDW is an indication of which of the following:
a. Variability of RBC volume
b. Poikilocytosis
c. Macrocytosis
d. Microcytosis
Feedback: Red Cell Distribution Width (RDW) is a mathematical expression of size variation used
toquantify anisocytosis. The higher the RDW, the greater the anisocytosis. RDW is increased
iniron deficiency, and tends to be normal in thalassemia. Increased RDW may be an early
indication of iron deficiency, where it may precede the onset of microcytosis.
8. Group A beta-hemolytic streptococci are best characterized by which of the following:
a. Positive cAMP test
b. Optochin sensitivity
c. Bile esculin-hydrolysis
d. Bacitracin sensitivity
Feedback: Group B streptococci are characterized by a positive cAMP test. Streptococcus
pneumoniae is characterized by Optochin susceptibility Enterococci are positive by the bile-
esculin test.
9. The turbidity of the bacterial inoculum of a standard disc diffusion (Kirby-Bauer)susceptibility
test is:
a. McFarland Standard
b. 0.5 McFarland Standard
c. McFarland Standard
d. McFarland Standard
10. Which of the following is not a common support medium used in electrophoresis techniques:
a. Cellulose acetate
b. Agarose gel
c. Polyacrylamide gel
d. Dextrose
Feedback: Cellulose acetate and agarose gels are most commonly used in the clinical laboratory
11. The normal range for urine pH is:
a. 4.6 to 8.0
b. 5.0 to 6.0
c. 7.0 to 8.0
d. none of the above
12. Which of the following is not a likely cause of an abnormal thrombin time (TT):
a. Fibrin split products
b. Heparin
c. Aspirin
d. Dysfibrinogenemia
Feedback: The thrombin time is performed by adding thrombin to citrated plasma. It is
prolonged by heparin, a deficiency or abnormality of fibrinogen and the presence of fibrin split
products.Aspirin affects primarily platelet function.
13. What is the eight hour occupational exposure limit for a chemical called?
a. Threshold limit value
b. Threshold time limit
c. Time limit value
d. Short term limit
14. Spherocytes are associated with which two of the following conditions:
a. Hereditary spherocytosis
b. Autoimmune hemolytic anemia
c. Thalassemia
d. Iron deficiency
Feedback: Spherocytes result from removal of small amounts of erythrocyte membrane, with
resultant reduction in surface to volume ratio. This may occur in the reticuloendothelial system,
as inhereditary sherocytosis and autoimmune hemolytic anemia, or may be due to direct injury
of red cells, as in patients with severe burns.
15. The generally accepted age range for homologous blood donation is:
a. 21 – 65 years
b. 17 – 65 years
c. 15 – 65 years
d. Over the age of 14years
Feedback: Prospective donors over 65 may be accepted at the discretion of the blood bank
physician.Many donor centers safely involve elderly donors in their collection programs.
16. Which of the following organisms in most frequently associated with endocarditis:
a. Viruses
b. Neisseria
c. Staphlyococci
d. Streptococci
Feedback: Various Streptococci are the most common causes of endocarditis, although many
otherorganisms have been implicated.
17. Ionized calcium is most commonly measured using which of the following method:
a. Flame photometry
b. Color complex formation between calcium and o-cresolphthalein
c. Atomic absorption
d. Calcium ion selective electrodes
18. Match the clotting factor with its commonly associated name:
Factor II - Prothrombin
Factor I - Fibrinogen
Factor V – Proaccelerin
19. The prozone effect ( when performing a screening titer) is most likely to result in:
a. False positive
b. False negative
c. No reaction at all
d. Mixed field reaction
Feedback: Prozone effect (due to antibody excess) will result in an initial false negative in spite
of the large amount of antibody in the serum, followed by a positive result as the specimen is
diluted.
20. Which of the following tests on amniotic fluid would be included when assessing fetal maturity:
a. Alpha fetoprotein and bilirubin
b. L/S ratio and bilirubin
c. Alpha fetoprotein and L/S ratio
d. Creatinine and L/S ratio
21. Therapeutic hemapheresis may be used to treat all of the following except:
a. Acute Guillian-Barre syndrome
b. Paraproteinemia
c. HIV infections
d. Cryoglobulinemia
Feedback: Therapeutic Hemapheresis is the process of removing a harmful blood component,
and returned the remaining blood components to the patient. The examples cited above are
indications for plasmapheresis, in which plasma containing a harmful substance is removed from
the body, and replaced with FFP or other volume expanders, depending on the patient's
condition.
22. A smear that is prepared from equal parts of methylene blue and whole blood will be used for:
a. WBC differential
b. Platelet estimates,
c. RBC count
d. Reticulocyte count
Feedback: Remaining nuclear fragments found in reticulocytes take up the methylene blue dye.
23. Gram positive organisms
a. resist acetone-alcohol de colorization
b. are always decolorized by acetone-alcohol
c. resist staining by crystal violet
d. readily stain with safranin in the Gram stain
Feedback: Gram positive organisms resist decolorization with acetone-alcohol. Gram negative
organismsare decolorized, and subsequently stain with safranin.
24. Which of the following are not appropriate indications for the use of fresh frozen plasma:
a. Volume expansion
b. Severe bleeding in the presence of significantly elevated PT secondary to vitamin K
deficiency
c. Replacement component after plasma exchange in patients with TTP or HUS
d. Dilutional coagulopathy with significant active bleeding, and PT and PTT twice normal
Feedback: Since each unit of fresh frozen plasma (FFP) carries with it the risk of transmitting
blood bornepathogens, and other products which do not carry this risk are available, FFP should
not be used as a volume expander
25. If a pipette is labeled (TC) " to contain " you would do the following:
a. Drain pipette, but not blow out
b. Drain contents then blow out or rinse
c. Drain to last mark on pipette
d. Do not consider the meniscus when filling
Feedback: "TC" means the total volume contained in the pipette - you must blow out to remove
total contents.
26. The concentration of sodium chloride in an isotonic solution is :
a. 8.5 %
b. 0.85 %
c. 0.08 %
d. 1molar
Feedback: Isotonic or normal saline is a 0.85 % solution of sodium chloride in water.
27. The positive square root of the variance of a set of values is called:
a. Median value
b. Mean value
c. Standard deviation
d. Coefficient of variation
28. Patients with antibody to the following antigen are immune to Hepatitis B:
a. Core antigen
b. Surface antigen
c. e antigen
d. Delta antigen
29. Which one of the following is not a true statement about Chlamydia:
a. Requires tissue culture for growth
b. Is a true bacterium
c. Is an obligate intracellular parasite
d. Has the nuclear structure of a virus
30. Which of the following species are gram positive:
a. Streptococcus
b. Neisseria
c. Listeria
d. Lactobacillus
31. Which of the following media is commonly used when performing the Kirby-Bauer disk diffusion
a. Chocolate agar
b. Mueller-Hinton agar
c. Thayer-Martin agar
d. MacConkey agar
32. Which of the following best defines "sensitivity":
a. The percentage of patients with a disease who will have a positive test
b. The percentage of patients without a disease who will have a negative test
c. The percentage of patients with a disease who will have a negative test
d. The percentage of patients without a disease who will have a positive test
Feedback: Sensitivity refers to the percentage or fraction of patients with a given disease in a
givenpopulation who will have a positive test.
33. What percentage solution of sodium hypochlorite (bleach) is recommended as a routine
laboratory disinfectant:
a. 5 %
b. 10%
c. 15%
d. 20%
34. In HDN which of the following antigen-antibody reactions is occurring:
a. Maternal antibody against fetal antibody
b. Maternal antigen against fetal antibody
c. Maternal antibody against fetal antigen
d. Maternal antigen against fetal antigen
35. Carbon dioxide is predominately found in blood in the form of:
a. Bound CO2
b. Bicarbonate ions
c. Sodiumcarbonate
d. Pco2
36. What condition is indicated by the following blood gas results: Bicarbonate = 32 mmol/L(Normal
= 22 - 26 mmol/L); pCO2 = 65 mm Hg (Normal = 35 - 45 mmHg); pH = 7.28(normal = 7.35 - 7.45)
a. Healthy condition
b. Uncompensated metabolic acidosis
c. Compensated metabolic acidosis
d. Uncompensated respiratory acidosis
Feedback: The normal pH of blood is 7.40. In order for most metabolic processes to take place,
the pH must remain within a narrow range close to this value. The range is usually defined in
adults as 7.35-7.45. If blood pH falls below 7.35, the blood becomes too acidic (acidosis). If blood
rises above 7.45, the blood is too alkaline (alkalosis).As blood pH decreases, the kidneys will
retain bicarbonate (HCO3-) from the glomerular filtrate; therefore, bicarbonate is increased.
However, in this case, the increased HCO3- could not compensate for the markedly elevated
pCO2 (the respiratory component) and the condition that results is uncompensated respiratory
acidosis.
37. In a normal CSF the protein concentration as compared to that in the serum is generally:
a. Less than50%
b. Less than30%
c. Less than10%
d. Less than 1%

Feedback: Normal ratio of serum to CSF Protein is about 200 to 1, or less than 1%. Elevation
of CSF total protein is a nonspecific finding, but usually indicates some type of disease
involving the brain or meninges.

38. Match the type of hepatitis with its route of transmission

Fecal and Oral - HepatitisA

Parenteral - HepatitisB

Parenteral - HepatitisC

Feedback: Hepatitis A is spread through the fecal-oral route. Hepatitis B is most commonly
spread throughparenteral means, but nonparenteral routes such as transmission through
pregnancy, sexualcontact, and exposure to infected body fluids are also important. Hepatitis
C is spread throughparenteral routes; pregnancy, sexual contact, and in households is
possible, but much lesslikely than with hepatitis B.
39. Which of the following staining methods would you use to demonstrate the metachromatic
granules of Corynebacterium diphtheriae:
a. Acid fast
b. Gram
c. Trichrome
d. Methyleneblue
Feedback: Methylene blue stain can be used to visualize the metachromatic granules that are
characteristic of C. diphtheriae.
40. Which of the following are not considered normal flora of the gastrointestinal tract:
a. Lactobacillus
b. Clostridium
c. Peptostreptococcus
d. Shigella
Feedback: Shigella and Salmonella are not found as normal flora in the gastrointestinal tract.
41. All of the following cellular antigens are important to an immunohematologist except:
a. Blood group antigens
b. Histocompatibility antigens
c. Haptens
d. Autoantigens
Feedback: A hapten is an incomplete antigen.
42. Which one of the following statements about anti-A1 is false:
a. Anti-A1 is a naturally occurring antibody that is usually clinically insignificant
b. Anti-A1 reactive at 37oC can sometimes destroy transfused A1 positive cells
c. A1 and A2 occur in 80 and 20 percent of the population, respectively
d. A2 cells react with Dolichos biflorus lectin
Feedback: Dolichos biflorus lectin has anti-A1 specificity.
43. Which one of the following organisms do not usually stain with an acid-fast stain, or one of its
variants:
a. Mycobacterium
b. Cryptosporidium
c. Actinomyces israelii
d. Nocardia
44. Which of the following would be the most appropriate temperature for long term storage of
viral cultures:
a. 4° C
b. -20° C,
c. -70° C
d. Room temperature
Feedback: Cultures can be stored at either -20° or -70°, but -70° is preferred for long term
storage.
45. Which of the following would be the most appropriate method to confirm a positive protein
from a urine dipstick: Immunoelectropheresis
a. Heat precipitation
b. Sulfosalicylic acid precipitation
c. Protein electrophoresis
46. Circulating organic iodine is found primarily in the form of:
a. Triiodothyronine
b. Parathyroidhormone
c. Thyroglobulin
d. Thyroxine
Feedback: Iodine circulates as thyroxine (T4) and to and to a lesser extent as Triiodothyronine
(T3).
47. Which of the following conditions would be suggested by a marked rise in alkalinephosphatase,
jaundice, and a moderate rise in ALT:
a. Cardiovascular disease
b. Hemolytic anemia
c. Post-hepatic obstruction
d. Renal failure
Feedback: Post-hepatic obstruction is characterized by a marked increase in conjugated
bilirubin.
48. Which of the following organisms is the most common cause of acute cystitis:
a. Escherichia coli
b. Klebsiella pneumonia
c. Chlamydia trachomatis
d. Staphylococcus epidermidis
49. A fluorometer operates on which of the following principles:
a. Measures amount of light absorbed by a given compound
b. Measures changes in emitted wavelength of light after absorption by an analyte
c. Measures amount of light scattered by the analyte in question
d. Measures light emitted at two or more wavelengths
50. Which of the following statement about synovial fluid is true:
a. Markedly elevated neutrophils always correspond to bacterial joint infection
b. Patients with gout involving joints will have markedly elevated neutrophils in their
synovial fluid
c. Fungal joint infections have synovial fluid with numerous lymphocytes
d. Tuberculous joint infections have synovial fluid with numerous plasma cells
51. Rheumatoid factor reacts with which the following :
a. Fc fragment of IgM
b. Fab fragment of IgM
c. Fab fragment of IgG
d. Fc fragment of IgG
52. Gamma irradiation of cellular blood components is required in which of the following situations:
a. Prevent post-transfusion purpura
b. Prevent Graft-Versus-Host (GVH) disease
c. Sterile components
d. Prevent non-cardiogenic pulmonaryedema
Feedback: Transfusion related Graft-Versus-Host disease is a rare condition usually following
transfusion of patients whose immune system is severely compromised. It occurs when T
lymphocytes present in the transfused unit replicate and attack the tissues of the recipient.
Gamma irradiation prevents this condition by inactivating T lymphocytes in the donor unit.
Gamma irradiation of cellular blood components is required when: 1.The donor is a blood
relative of the recipient, 2. intrauterine transfusions, 3. The recipient has a selected
immunodeficiency condition, 4. The recipient has received a bone marrow transplant.
53. Patients with which of the following conditions would benefit most from washed red cells:
a. Warm autoimmune hemolytic anemia
b. Cold autoimmune hemolytic anemia
c. Elevated serum potassium
d. Multiple red cell alloantibodies
Feedback: Washing not only reduces the number of leukocytes and platelets that are often
responsible for febrile reactions, but also eliminates anticoagulants, ammonia, lactic acid, and
potassium.
54. Cryoglobulin testing can be used to:
a. Screen for rheumatoid arthritis,
b. Diagnose syphilis
c. Screen for systemic lupus erythematosus (SLE)
d. Help diagnose Raynaud's syndrome
Feedback: Cryoglobulin is an abnormal serum protein with a unique ability to precipitate when
exposed to cold temperatures. Cryoglobulin is present in Raynaud's syndrome.
55. Which is arranged from least mature to most mature:
a. Myeloblast, Myelocyte, Band, Neutrophil, Metamyelocyte, Promyelocyte, Stem Cell
b. Stem Cell, Myeloblast, Promyelocyte, Myelocyte, Metamyelocyte, Band, Neutrophil
c. Stem Cell, Promyelocyte, Metamyelocyte, Neutrophil, Myeloblast, Myelocyte,Band
d. Neutrophil, Band, Metamyelocyte, Myelocyte, Promyelocyte, Myeloblast, StemCell
56. The measurement of total glycosylated hemoglobin A1c is an effective means of assessing the
average blood glucose levels:
a. Over the past 1-2weeks
b. Over the past 2-3months
c. Only the past 24 hours
Feedback: The concentration of hemoglobin A1c is directly proportional to the average plasma
glucose level that the RBCs are exposed to during their 120 day life span. The glycosylated
hemoglobin result from glycosylation of serine, asparagine, and hydroxylysine residues in
hemoglobin. There are three glycosylated hemoglobins: Hb A1a, Hb A1b, and Hb A1c, of which
Hb A1c constitutes 80%. Most methods measure total gycosylated hemoglobin, which has a high
degree of correlation to Hb A1c, and may be reported as such after making the appropriate
conversion.
57. Bacitracin susceptibility is useful for which two of the following:
a. Distinguishing Staphylococci from micrococci
b. Presumptive identification of Group A streptococci
c. Identification of Haemophilus
d. Identification of Neisseria
Feedback: Staphylococci are resistant to bacitracin while micrococcus are sensitive. Group A
beta-hemolytic Streptococci are sensitive to bacitracin, while most other Groups are resistant.
58. Which of the following is the proper temperature to use when crossmatching in the presence of
a cold antibody:
a. 37 degreesCelsius
b. 25 degreesCelsius
c. 15 degreesCelsius
d. 4 degrees Celsius
Feedback: Most antibodies that are inactive at 37 degrees Celsius, and active only below 37
degreesCelsius (i.e. cold reactive antibodies), are of little clinical significance.
59. Match the virus with its disease:
Hepadnavirus - Hepatitis
Arbovirus - Encephalitis
Parvovirus - Fifth disease
Paramyxovirus -Measles-Mumps
60. When a urine specimen cannot be cultured immediately it may be:
a. Stored at room temperature for up to 8hours
b. Stored in freezer for up to 24 hours
c. Neisseria
d. Stored at 4° C for up to 24 hours
61. Platelet should be stored at what temperature :
a. 1-6 degrees Celsius
b. 10-16 degreesCelsius
c. 20-24 degreesCelsius
d. 34-37 degreesCelsius
Feedback: Platelets should be stored at 20-24 degrees Celsius with continuous gentle agitation.
They should be infused within 4 hours after the seal on a platelet unit is broken.
62. The recommended temperature/time/pressure for routine steam sterilization of media is:
a. 121° C for 8-10 minutes @ 10psi
b. 121° C for 12-15 minutes @15 psi
c. 220° C for 8-10 minutes @ 10psi
d. 220° C for 12-15 minutes @15 psi
63. Which of the following casts might be found in the urine of a healthy individual after strenuous
exercise:
a. Epithelial cell casts
b. Hyaline casts
c. Granular casts
d. Waxy casts
Feedback: Hyaline casts may be seen in healthy individuals particularly after exercise.
64. If a laboratory 's control range (using a 99.7 confidence interval) for a given assay is 20.0 to50.0,
what would it’s means and one standard deviation be:
a. Mean = 35.0 SD =15.0
b. Mean = 35.0 SD =10.0
c. Mean = 40.0 SD =7.5
d. Mean = 35.0 SD =5.0
Feedback: The mean would be halfway between 20 and 50, or 35. The range for the assay is
within 15units of the mean, representing 3 SD (equal to 99.7% confidence interval). Therefore, 1
SD =15/3 or 5.
65. If a potential donor has been transfused blood products, he must be deferred from blood
donation for:
a. 1 month
b. 3months
c. 6months
d. 1 year
66. An urticarial reaction is characterized by:
a. Rapid rise in temperature
b. Difficulty breathing
c. Rash and hives
d. Blood in the urine
Feedback: Urticarial transfusion reactions occur in up to 1% of transfusions. They are
manifested by hives, rash, and itching. The transfusion may be resumed after successful
administration of antihistamines.
67. Which of the following cells when found upon microscopic examination of the urine would be
most indicative of kidney disease:
a. WBCs and bacteria
b. Tubular epithelial cells
c. Squamous epithelial cells
d. RBCs
Feedback: Squamous epithelial cells are usually vaginal contaminants.
68. Which of the following statistical methods would be employed to determine how closely two
different methods compare with each other:
a. Linear regression
b. Standard deviation
c. Coefficient of variation
d. Percent difference of means
Feedback: Linear regression can be used to compare one method with another.
69. Elevation in CSF total protein may be seen in all of the following conditions except:
a. Brain tumors
b. Bacterial meningitis
c. Parkinson'sdisease
d. CVA
70. Which one of the following is not a system for incubation of anaerobic bacteria:
a. Candle jar
b. Glove box anaerobic chamber
c. Anaerobic jar – gaspak
d. Anaerobic jar – evacuation replacement
71. A plastic anemia may be caused by all of the following except:
a. Infections
b. Chemical agent
c. Enzyme deficiencies
d. Ionizing radiation
Feedback: Enzyme deficiencies are usually associated with hemolytic anemias.
72. Which of the following types of whole blood would be the least satisfactory to transfuse to a
type AB patient:
a. Group O
b. Group A
c. Group B
d. GroupAB
Feedback: Group O whole blood contains both anti-A and anti-B which could react with the
recipient'sRBCs.
73. Bence-Jones proteinuria is associated with all of the following conditions except:
a. Macroglobulinemia
b. Pernicious anemia
c. Multiple myeloma
d. Amyloidosis
Feedback: Macroglobulinemia, Multiple Myeloma, and Amyloidosis are all associated with
abnormal protein production and/or accumulation which can result in Bence-Jones proteins
being excreted in the urine.
74. The slide coagulase test is a rapid method for identifying which of the following organisms.
a. Staphylococcus hominis
b. Staphylococcus pneumonia
c. Staphylococcus aureus
d. Neisseria gonorrhoeae
Feedback: Since not all isolates of S. aureus are detected by the slide coagulase test, suspicious
organisms negative by the slide test must be confirmed by the tube coagulase test.
75. Which of the following streptococcus exhibits a positive bile solubility test:
a. S. pyogenes
b. S.pneumoniae
c. S. agalactiae
d. S. mutans
Feedback: Streptococcus pneumoniae colonies dissolve with the addition of sodium
deoxycholate (positive bile sensitivity test) and are sensitive to Optochin- unlike other species of
Streptococci.
76. Which of the following red blood cells contain the most H antigen:
a. Group A cells
b. Group B cells
c. Group O cells
d. Bombayphenotype
Feedback: O cells contain only H antigen, while cells of the other ABO groups contain varying
amount of H antigen in addition to their group specific antigen(s). The classic Bombay
phenotype is characterized by the absence of A, B, and H antigens.
77. A zinc deficiency in the elderly is often caused by:
a. Decreased intake and absorption
b. Decreased intake and excretion
c. Increased intake and excretion
d. Increased excretion and decreased absorption
78. Match the illustrations with the corresponding description of colony edges:

Irregular - A
Entire – B
Crenated – C
79. ABO blood groups were discovered by:
a. Mendelson
b. Morgan
c. Wiener
d. Landsteiner
Feedback: Landsteiner first identified the presence of the separate red cell antigens A and B in
the early1900's.
80. Which of the following parasites is not commonly found in the peripheral blood:
a. Loa loa
b. Brugia malayi
c. Dipetalonema perstans
d. Onchocerca volvulus
Feedback: The microfiliariae of Onchocerca do not generally circulate in the peripheral blood.
The diagnosis is made by demonstrating the microfiliariae in teased skin snips.
81. If an Rh negative patient is administered a unit of R1R1 packed red cells, which one of the
following antibodies would be most likely to develop:
a. Anti-c,
b. Anti-E
c. Anti-D
d. Anti-e
Feedback: R1R1 (DCe/DCe) cells are positive for the D antigen, which is the most immunogenic
antigen of the Rh system, followed by c and E.
82. A decrease in which of the following in AIDS patients results in increased susceptibility to
infection:
a. CD 4+ cells
b. CD 8+ cells
c. HIV antibodies
d. HIV antigens
Feedback: It is the CD 4+ cells that help aid our immune system in attacking foreign antigens by
initiating the body's response. CD 8+ also helps in the role of immunity by attaching to infected
cells to initiate cell destruction. A decrease in HIV antibodies would not leave the host more
susceptible to infection since the antibody is directed at the HIV virus only. Finally, a decrease in
HIV antigens will also not cause a host to become more suseptible to infections.
83. The primary site of reabsorption of glomerular filtrate is the:
a. Loop of Henle
b. Proximal convoluted tubule
c. Distal convoluted tubule
d. Collecting duct
84. Which of these is arranged from least mature to most mature:
a. Lymphoblast, Lymphocyte, Prolymphocyte, StemCell
b. Stem Cell, Lymphoblast, Prolymphocyte, Lymphocyte
c. Stem Cell, Lymphoblast, Lymphocyte,Prolymphocyte
d. Lymphocyte, Prolymphocyte, Lympoblast, StemCell
85. MCV is calculated using which of the following parameters:
a. Hgb and RBC
b. RBC histogram
c. RBC and Hct
d. RBC and MCHC
Feedback: MCV = Hematocrit X 1000/RBC (in millions per microliter). The MCV is expressed in
femtoliters (cubic micrometers, 10-15 liters).
86. Which of the following would not be considered a normal part of a routine electrolyte panel:
a. Sodium
b. Potassium
c. Magnesium
d. Chloride
Feedback: A routine electrolyte panel typically consists of sodium, potassium, chloride,
bicarbonate, creatinine, glucose, and BUN.
87. Which of the following conditions is associated with elevated serum uric acid levels:
a. Cushing'ssyndrome
b. Pancreatitis
c. Hyperthyroidism
d. Gout
Feedback: Gout is a disease manifested by the deposition of crystalline uric acid in joints and
soft tissues,with secondary inflammation and pain.
88. Which of the following additives should be used for the collection of a sample for blood gas
analysis:
a. Sodium citrate
b. EDTA
c. Sodium oxalate
d. Heparin
89. What is the site used most frequently in the adult patient when performing a bone marrow
biopsy:
a. Vertebrae
b. Sternum
c. Clavicle
d. Iliac crest
Feedback: Bone marrow specimens are usually obtained in adults from the posterior iliac crest.
They may also be obtained from the sternum.
90. The precursor of the platelet which is commonly only found in the bone marrow is:
a. Myeloblast
b. Metakarocyte
c. Megakaryocyte
d. Erythroblast
Feedback: Platelets are formed by the fragmentation of megakaryocyte cytoplasm.
91. Which of the following set of conditions would preclude HDN as a result of Rh incompatibility:
a. Mother Rh-negative, father Rh-positive
b. Mother Rh-negative, baby Rh-positive
c. Mother Rh-negative, father Rh-negative
d. Mother Rh unknown, baby Rh-positive
Feedback: If both parents are Rh negative, the baby would also be Rh negative.
92. Erythropoietin is produced in:
a. Bone marrow
b. Liver
c. Lymphocytes
d. Kidneys
Feedback: Erythropoietin, which is produced in the kidneys, stimulates the production of red
blood cells.
93. The Rh nomenclature which uses the letters DCE is found in which of the following genetic
models:
a. Landsteiner
b. Wiener
c. Fisher-Race
d. Rhesus
Feedback: Fisher and Race first proposed the existence of the three closely linked genes, and
used the DCE terminology to describe their theory.
94. Which of the following organisms is an obligate intracellular parasite:
a. Proteus
b. Escherichia
c. Klebsiella
d. Rickettsiae
95. An average adult would excrete approximately what volume of urine per 24 hours:
a. 3000ml
b. 1500ml
c. 750 ml
d. 250 ml
Feedback: Normal adult urine volume is 800 to 1800 ml/24hours for males, and 600 to 1600
ml/24 hours for females.
96. What minimum level of specific resistance (megohms@25oC) is required for a Type I
watersystem:
a. 10.0
b. 5.0
c. 2.0
d. 0.1
Feedback: Specific resistance is the resistance in Ohms of a 1 cm long by 1 cm2 cross sectional
column of solution. Type I water is the purest, and therefore has the highest resistance. Type II
water is used for most routine laboratory determinations, and must have a specific resistance of
at least 2.0. Type III water is the least pure; it must have a specific resistance of at least 0.1.
97. Which of the following group B antigens is generally associated with a mixed field reaction:
a. B
b. B3
c. Bm
d. Bx
Feedback: B3 is characterized by a weaker than usual reaction with anti-B and by a mixed field
reaction with the same reagent.
98. The most common rapid slide test (MONOSPOTâ) for infectious mononucleosis employs:
a. Horse erythrocytes
b. Sheep erythrocytes
c. Intact beef erythrocytes
d. None of the above
Feedback: A positive spot test shows agglutination of horse erythrocytes when added to patient
serum previously absorbed with guinea pig kidney, but not when added to patient serum
previously absorbed with beef erythrocyte stroma. The test is generally simple, sensitive, and
specific, but false negative are common in young children with mononucleosis.
99. Disseminated intravascular coagulation (DIC) is associated with all of the following clinical
conditions except:
a. Septicemia
b. Obstetric emergencies
c. Intravascular hemolysis
d. Thrombocytosis
Feedback: Platelet counts are decreased or normal during DIC.
100. Which of the following organisms is most likely to be associated with gas gangrene:
a. Clostridia histolyticum
b. Pseudomonas aeruginosa
c. Clostridium perfringens
d. Escherichia coli
101. Which of the following would most likely occur as the result of hemodilution:
a. Low serum electrolytes
b. High serum electrolytes
c. Increased anion gap
d. Electrolyte concentrations would remain the same
Feedback: A dilutional effect caused by the sample being hemodiluted, would not cause high
serum electrolytes since they have been diluted out. The anion gap could remain the same or
become decreased. Finally, the electrolyte concentrations could not remain the same, since
there will be a smaller amount in the sample to test due to the diluent.
102. Following a myocardial infarction which of the following enzymes will be the first to
become elevated:
a. CK
b. LDH
c. GGT
d. AST
Feedback: GGT is elevated in liver disorders. CK rises before LDH. However myoglobin is the
earliest serum cardiac marker to rise after an infarction; it may be positive within two hours post
MI. It is not cardiac specific, and can be elevated in skeletal muscle trauma or rhabdomyolysis.
103. India Ink is used to:
a. Visualize flagella
b. Visualize shape
c. Visualize capsule
d. Visualize cytoplasm
Feedback: India ink can aid in the visualization of the polysacchride capsules of yeast such as
Cryptococcus neoformans.
104. Match the organisms on the right with their appropriate type from the drop down box:
Protozoa - Trypansoma,Leischmania
Helminths - Trichinella, Schistosoma
Protozoa - Plasmodium
Helminths - Ascaris, Cestodes
105. The most frequent genotype among Rho(D) -negative persons is:
a. r'r
b. r'r"
c. rr
d. r'r'
106. Which one tests should be used to monitor a patient's response to treatment for
syphilis:
a. TPI titer,
b. FTA-ABS titer
c. RPR titer
d. TPHA titer
Feedback: RPR titer should be performed every three months for one year; a fourfold increase
in titer indicates infection, reinfection, or failed treatment; a fourfold decrease in titer indicates
adequate treatment.
107. Which of the following anticoagulants will not produce a significant effect on calcium
levels in plasma:
a. EDTA
b. Heparin
c. Oxalates
d. Citrates
Feedback: EDTA, Oxalates, and Citrates remove calcium from the blood, while heparin acts by
forming a complex with antithrombin III.
108. Which of the following kappa / lambda ratios is found in normal serum:
a. 4 :1
b. 3 :1
c. 2 :1
d. 1 :1
109. The A1 subgroup represents approximately what percentage of group A individuals:
a. 25%
b. 55%
c. 75%
d. 95%
Feedback: Approximately 80% of all people typed as group A or AB are agglutinated by anti- A1
sera.
110. The infective stage for all of the intestinal amoebae is the:
a. Trophozoite
b. Cyst
c. Larva
d. Egg
Feedback: The majority of the amoebae pass through two life cycle stages: a delicate feeding
stage call a trophozoite, and a non-feeding infective cyst. Recovery of either form is considered
diagnostic.
111. Which of the following is false about troponin:
a. It rises much sooner after an MI than CK-MB
b. It stays positive much longer than CK-MB
c. It is potentially more specific than CK-MB
d. Troponin T is often elevated in renal failure patients
Feedback: Troponin is potentially more specific for myocardial damage than CK-MB and stays
elevated longer. It may eventually replace CK-MB as the standard marker of myocardial damage.
Troponin and CK-MB both tend to rise approximately 3 hours after a MI; however, troponins can
stay elevated up to 2 weeks as CK-MB tends to return to baseline around 36 hours.
112. What is the purpose of using methyl alcohol in the gram stain procedure:
a. Fix all gram positive organisms
b. Decolorize all gram positive organisms
c. Decolorize all gram negative organisms
d. Fix all gram negative organisms
Feedback: Gram positive organisms resist decolorization by methyl alcohol.
113. Match the differential/ selective enteric medium with its characteristic indicator,
fermentable, and bacteriostatic.
Xylose-lysine-deoxycholate - phenol red, lactose, bile salts
Eosin methylene blue - eosin Y and methylene blue, lactose, eosin Y and methyleneblue
Bismuth sulfite - bismuth sulfite, glucose, brilliant green
Salmonella-shigella - neutral red, lactose, bile salts
Feedback: XLD has a yellow fermenter and a red nonfermenter. MacConkey has a red fermenter
and a colorless nonfermenter. EMB has a black/red fermenter and a colorless nonfermenter. HE
has a yellow fermenter and a green nonfermenter. BS has a Salmonella fermenter and a black
nonfermenter. SS has a red fermenter and a colorless nonfermenter.
114. Nephelometry involves the measurement:
a. Light absorption
b. Light transmission
c. Light scatter
d. Atomic absorption
Feedback: Nephelometry is used to measure the light scattered by particles in a solution. It is
useful for measuring protein levels in fluids, and antigen-antibody complexes.
115. Which of the following procedures would you employ to monitor the precision of an
assay:
a. Run assayed controls
b. Run blinded samples
c. Run by alternative method
d. Run samples in duplicate
Feedback: Precision is a measure of reproducibility of a test when it is repeated several times
under the same conditions. It could be monitored by running samples in duplicate or triplicate.
116. Which of the following immunoglobulin classes is chiefly responsible for the
degranulation of mast cells and basophils:
a. IgG
b. IgA
c. IgM
d. IgE
Feedback: IgE levels are often increased in patients with allergic disease. IgE binds to the
membranes of mast cells and basophils, and if specific antigen is present to react with the IgE
molecule, degranulation of these cells occurs, releasing histamines, and other substances into
the blood or tissues.
117. The lecethin to sphingomyelin ratio (L/S) is used to assess:
a. Fetal neurological development
b. Fetal lung maturity
c. Fetal viability
d. Fetal liver development
Feedback: An L/S ratio above 2:1 indicates mature lung development, whereas a ratio below
1.5:1indicates pulmonary immaturity.
118. What additional fraction would be seen if plasma rather than serum was subjected to
electrophoresis:
a. Alpha-1 antitrypsin,
b. Gammaglobulins
c. Alpha-2 macroglobulin
d. Fibrinogen
Feedback: Fibrinogen present in plasma would form a band between the beta and gamma
globulins.
119. Which of the following is a routine staining technique used in the clinical laboratory:
a. Wright-Giemsa
b. Gram
c. Romanowsky
d. All of the above
120. Match the description with the appropriate illustration of colony elevations:

High Convex A
Flat B
Low Convex C
Plateau D
121. Which of the following is most commonly associated with febrile non-hemolytic
transfusion reactions:
a. Bacterial contamination of the blood
b. I.V. tubing contaminants
c. Reaction to plasma proteins
d. Immune response to leukocytes
122. Which of the following is not a structural component of a typical virion:
a. Nucleoprotein core
b. Capsid shell
c. Lipid envelope
d. Icosahedral symmetry
Feedback: An intact viral particle typically consists of a nucleoprotein core, capsomeres making
up the capsid, and a lipid envelope (absent in some viruses). The symmetry of a virus is usually
icosahedral, or helical.
123. Match urine color with substance that might have been responsible:
Phosphates- White
Porphobilinogen- Red tobrown
Bilirubin- Yellow
Pseudomonas- Blue togreen
124. The renal threshold is best described as:
a. Concentration at which a substance in the blood spills into urine
b. Concentration at which reabsorption first occurs
c. Concentration at which kidney can no longer filter the blood
d. Concentration at which kidney failure begins
Feedback: The renal threshold is the maximum amount of a substance that the kidney can
prevent from entering into the urine.
125. An urticarial reaction is characterized by:
a. Rapid rise in temperature
b. Difficulty breathing
c. Rash and hives
d. Blood in the urine
Feedback: Urticarial transfusion reactions occur in up to 1% of transfusions. They are
manifested by hives,rash, and itching. The transfusion may be resumed after successful
administration of antihistamines.
126. On sheep blood agar Haemophilus influenzae may exhibit satellite formation around all
but which of the following organisms:
a. Pseudomonas
b. Neisseria
c. Staphylococcus
d. Pneumococcus
Feedback: Neisseria, Staphylococcus, and Pneumococcus produce enough V factor to allow H.
influenza to grow on blood agar near their colonies. This property is utilized in the "satellite
test" technique whereby minute colonies of Haemophilus are seen in the hemolytic zone
surrounding a streak of Staphylococcus aureus on sheep blood agar, providing a presumptive
identification of Haemophilus.
127. Adult Hematopoiesis occurs in:
a. Vertebrae
b. Skull
c. Proximal ends of long bones
d. All of the above
Feedback: By the age of 18-20 years, active hematopoiesis is limited to the sternum, ribs, pelvis,
vertebrae, and skull.
128. A simple check which can be employed to verify that hemoglobin and hematocrit values
match would be:
a. Hematocrit X 3 =hemoglobin
b. Hemoglobin X 3 =hematocrit
c. Hemoglobin / hematocrit =3
d. Hemoglobin + hematocrit =3
Feedback: The hematocrit is equal to approximately 3 times the hemoglobin level.
129. How close the assayed value of an analyte is to its actual value is a reflection of:
a. Precision of the assay
b. Reproducibility of the assay
c. Sensitivity of the assay
d. Accuracy of the assay
Feedback: Accuracy is defined as how close an assayed value is to the actual value of an analyte.
130. The primary mechanism responsible for glomerular filtration is:
a. Osmotic gradient
b. Concentration of blood components
c. Rate of blood flow through the kidneys
d. Hydrostatic differential in glomerular tufts
Feedback: The hydrostatic pressure in the capillaries of the glomerular tuft drives the filtrate
across their semipermeable membrane. The normal glomerular filtrate is similar in composition
to the plasma, with the exception that molecules with a molecular weight of greater than about
15,000 are not filtered.
131. Which of the following best describes a minor crossmatch:
a. Reaction of donor cells with recipient serum
b. Reaction of donor cells with AHG
c. Reaction of recipient cells with AHG
d. Reaction of donor serum with recipientcells
132. Fire requires what three elements?
a. heat, fuel, oxygen
b. paper, match, gasoline
c. heat, fuel, nitrogen
d. lighter fluid, charcoal,grill
133. A 20 year-old female was admitted into the hospital complaining of 10 to 15 bloody
mucous stools per day, fever, gastrointestinal disturbances, abdominal pain, and nausea. The
preliminary O & P report went out as "Probable Entamoeba histolytica trophozoites andcysts,
confirmation pending." This patient is most likely suffering from:
a. Traveler's diarrhea
b. Extraintestinal amebiasis
c. Intestinal amebiasis
d. Giardiasis
134. Which of the following Rh antigens is found the highest frequency in the Caucasian
population:
a. C
b. E
c. C
d. e
Feedback: The e antigen is present in 98% of the Caucasian population.
135. What is another name used to designate a fully committed B-lymphocyte:
a. T-lymphocyte
b. Reactive lymphocyte
c. Large lymphocyte
d. Plasma cell
Feedback: Plasma cells are the end stage of B lymphocyte maturation; they are not normally
seen in peripheral blood.
136. Serum calcitonin is typically elevated in which of the following conditions:
a. Medullary carcinoma of the thyroid
b. Hyperthyroidism
c. Glioblastoma
d. Adrenal adenoma
Feedback: Serum calcitonin is normally produced by the C cells of the thyroid. It functions to
reduce serum calcium by inhibiting release of calcium from bone. It is a peptide with a molecular
weight of 3400, and has a half-life of approximately 12 minutes. It is characteristically elevated
in medullary carcinoma of the thyroid. Since medullary carcinoma often occurs as an autosomal
disorder, family members of patients with this condition should be screened for serum
calcitonin.
137. Warfarin inhibits all the following coagulation Factors except:
a. Factor II
b. Factor IX
c. FactorVII
d. Factor XI
Feedback: Warfarin only inhibits vitamin K dependent factors; Factor XI is not vitamin K
dependent.
138. Match the organism names from the drop down box to the name of the associated
condition listed below.
Plasmodium ovale- Malaria
Trypanosoma cruzi- Chagas' Disease
Cestodes- Tapeworms
Trypanosoma rhodesiense- SleepingSickness
139. Serum proteins can be separated by cellulose acetate electrophoresis into how many
basic fractions:
a. 4fractions
b. 5fractions
c. 6fractions
d. 7fractions
Feedback: The five basic fractions are albumin, alpha-1, alpha-2, beta, and gamma globulins.
140. Which of the following specimens would not be considered suitable for anaerobic
culture:
a. Drainage from a puncturewound
b. Throat swab
c. Drainage from abdomen
d. Blood culture
Feedback: Anaerobes are predominant in the colon. Anaerobic habitats have reduced oxygen
tension usually resulting from the metabolic activity of aerobic organisms. Thus, anaerobic
infections are usually polymicrobial, with aerobes, facultative anaerobes, and obligate
anaerobes all contributing to the infection.
141. Which band on the following serum protein electrophoresis scan is not made up of a
mixture of proteins:
a. Beta
b. Alpha -1
c. Alpha -2
d. Albumin
142. Which of the following is not necessary for bacteria to grow successfully on artificial
media:
a. Pure isolate
b. Proper temperature
c. Proper Ph
d. Proper moisture
Feedback: A mixture of bacteria can easily grow together on media as long as the other factors
are all maintained correctly.
143. Why would a unit of group O blood never be administered to a Bombay patient:
a. Anti-A in donor
b. Anti-B in donor
c. Anti-H in donor
d. Anti-H in recipient
Feedback: The classic Bombay phenotype (Oh) is characterized by the absence of A, B and H
antigens, and the presence of anti-H, which will react from 4o to 37oC. Bombay patients must
therefore be transfused only with blood from donors of the Bombay phenotype.
144. Total magnification of a microscope can be determined by:
a. Multiplying power of objective times the power of the ocular
b. Dividing the power of objective by the power of the ocular
c. Dividing the power of the ocular by the power of the objective
d. Adding the power of the ocular and objective together X 10
145. IgM antibodies directed against red cells generally:
a. React best at 37 degrees Celsius
b. Cause severe hemolytic reactions
c. Are identified using the AHGtest
d. React best at room temperature
146. Identify the reaction seen in Illustration

a. Identity
b. Nonidentity
c. Partial identity
147. Which of the following organisms is not an aerobic organism:
a. Francisella
b. Bacteroides
c. Pseudomonas
d. Neisseria
Feedback: Francisella tularensis is an aerobic gram-negative coccobaccillary organism requiring
cystine or cysteine to grow. It causes tuleremia, which can manifest as cutaneous papules and
ulcers, conjunctivitis, fever, myalgias, and lymphadenopathy. It can be diagnosed by culture on
appropriate media, or serology. Bacteroides fragilis is an anaerobic gram-negative rod which is
resistant to most penicillins. It can cause a variety of life threatening infections. Pseudomonas
aeruginosa is a stictly aerobic gram-negative motile bacillus; it causes a variety of infections
especially among patients who are immunocompromised for a variety of reasons, and it causes
wound infections in moist air.
148. Which of the following statements about Rickettsia is false:
a. They are obligate intracellular parasites
b. They are gram negative bacilli
c. Associated diseases are usually diagnosed by serology
d. They are cultured in many hospital laboratories
Feedback: Various Rickettsial species cause typhus, Rocky Mountain spotted fever, Rickettsial
pox, and Q fever. Rickettsial organisms are cultured only by research and reference laboratories,
and the diagnosis is usually achieved by a combination of clinical findings, and acute and
convalescent serology. Early diagnosis of Rocky Mountain spotted fever can also be made by
immunoflourescence testing of a skin biopsy, where Rickettsia can often be found within
endothelial cells of capillary vessels.
149. Coarse basophilic stippling in all of the following EXCEPT:
a. Megaloblastic anemias
b. Thalassemias
c. Lead poisoning
d. Hemolytic anemia
Feedback: Hemolytic anemia may be associated with fine basophilic stippling due to
precipitated RNA in polychromatophilic erythrocytes. It is not usually associated with coarse
stippling. The other conditions above have coarse stippling in the setting of abnormal
hemoglobin synthesis.
150. Human chorionic gonadatropin (hCG) is used in the determination of:
a. Liver function
b. Fetal maturity
c. Pregnancy
d. Steroid levels
151. Estrogen and progesterone markers are most commonly used to provide prognostic
information about:
a. Breast cancer
b. Uterine cancer
c. Menopause
d. Cervical cancer
Feedback: Patients with tumors positive for both estrogen and progesterone receptors tend to
respond favorably to hormonal therapy, whereas those without generally do not. Patients with
positive estrogen and progesterone receptors also have a somewhat better prognosis.
152. Materials such as strong acids and bases are classified as:
a. Corrosives
b. Chemical irritants
c. Flammable liquids
d. Carcinogens
153. Elevation in conjugated bilirubin is most likely to be found in which of the following
conditions:
a. Transfusion reactions
b. Erythroblastosis fetalis
c. Cirrhosis of the liver
d. Biliary obstruction
Feedback: Posthepatic jaundice is the result of bile duct obstruction, not impaired hepatic
function. It will result in elevation of direct (conjugated) bilirubin.
154. Pluripotential stem cells are capable of producing which of the following:
a. Only T-lymphocyte and B-lymphocyte subsets
b. Erythropoietin, thrombopoietin and leukopoietin
c. Lymphoid and myeloid stem cells
d. Daughter cells from only a single cell line
Feedback: Pluripotential stem cells are ultimately capable of differentiating into all types of
leukocytes.
155. Match the differential/ selective enteric medium with its characteristic
indicator,fermentable, and bacteriostatic.
phenol red, lactose, bile salts
eosin Y and methylene blue, lactose, eosin Y and methylene blue
bismuth sulfite, glucose, brilliant green
neutral red, lactose, bile salts
Feedback: XLD has a yellow fermenter and a red nonfermenter. MacConkey has a red fermenter
and a colorless nonfermenter. EMB has a black/red fermenter and a colorless nonfermenter. HE
has a yellow fermenter and a green nonfermenter. BS has a Salmonella fermenter and a black
nonfermenter. SS has a red fermenter and a colorless nonfermenter.
156. What is the largest constituent of plasma non protein nitrogen:
a. Ammonia
b. Creatinine
c. Ketones
d. Urea
157. What principle(s) of flow cytometry are employed when performing
immunophenotyping:
a. Defraction gradients
b. Impedance
c. Defraction gradients and impedance
d. Flourescent antibody tagging and lightscatter
Feedback: Flow cytometry employs a combination of fluorescent antibody tagging of cells and
analysis with laser light scatter.
158. Which is arranged from least mature to most mature:
a. Stem Cell, Rubriblast, Prorubricyte, Rubricyte, Metarubricyte, Reticulocyte,Erythrocyte
b. Stem Cell, Prorubricyte, Metarubricyte, Erythrocyte, Rubriblast, Rubricyte,Reticulocyte
c. Erythrocyte, Reticulocyte, Metarubricyte, Rubricyte, Prorubricyte, Rubriblast,Stem Cell
d. Rubricyte, Rubriblast, Stem Cell, Prorubricyte, Erythrocyte, Reticulocyte,Metarubrictye
159. The term used to describe patients with absence of Rh antigens is:
a. Rhd
b. Rhnull
c. Rhmod
d. Rho
Feedback: Rhnull individuals have no Rh antigens. Rhmod individuals show reduced and varied
reactivity with Rh antigens.
160. Which is the best method for examination of synovial crystals:
a. Phase contrast
b. Darkfield microscopy
c. Brightfieldmicroscopy
d. Polarized light
Feedback: A polarizing microscope with a first order red compensator is used to identify crystals
in synovial fluid: Needle shaped monosodium urate crystals are strongly negatively birefringent,
while rhomboid calcium pyrophosphate crystals are weakly positively birefringent.
161. Which of the following antibodies is the most common cause of hemolytic disease of the
newborn:
a. Anti-A
b. Anti-B
c. Anti-E
d. Anti-D
162. Which of the following is not true about members of the Enterobacteriaceae:
a. Gram-negative
b. Oxidase positive
c. Reduce nitrate to nitrite
d. Ferment glucose
Feedback: Any oxidase-positive organism can be excluded from the Enterobacteriaceae. The
other characteristics are as a rule present in these organisms.
163. Match these autoimmune diseases with their corresponding serological markers:
Antimitochondrial antibodies
Anticentromere antibodies
Antineutrophilic antibodies
Antimyocardial antibodies
164. Match each virus with its appropriate nucleic acid content:
Togavirus
Coronavirus
Herpesvirus
Adenovirus
165. Which one of the following is not a benefit of using packed RBCs:
a. Decreasing the load of potential donor antibodies
b. Decreasing the risk of transfusion associated infection
c. Decreasing plasma volume
d. More efficient use of the whole blood unit
166. Match the following terms with the statement that best describes each:
Membrane covering the brain under the skull
Fluid accumulation due to a malfunction of ability to form and reabsorb fluid
Inflammation of the pleural membranes
Excess accumulation of fluid within any of the body's serous cavities
167. The adult worms of which of the following reside in the intestine or its blood vessels:
a. Heterophyes heterophyes
b. Schistosoma mansoni
c. Clonorchis sinensis
d. Schistosomahaematobium
168. Match the illustrations with the corresponding classification of bacteria:

169. If a test is said to have a sensitivity of 95%, it will :


a. Miss 5 out of 100 negatives
b. Miss 5 out of 100 positives
c. Detect 5 out of 100positives
d. Detect 5 out of 100negatives
Feedback: A sensitivity of 95% means that 5 results out of a possible 100 that should have been
detected as positive by a method will have been reported as negative. These results are termed
false negatives.
170. Which one of the following are not associated with a polyclonal (broadbased) increase
in gamma globulins?
a. Liver disease
b. Chronic inflammation
c. Immune reaction
d. Immunodeficiency
Feedback: Immunodeficiency would, of course, generally be associated with a decrease in
serum immunoglobulin levels, and an associated decreased gamma band.
171. Which of the following tests would be used to directly document the presence of a
specific organism in a clinical specimen:
a. ELISA test
b. Hemagglutination test
c. Hemagglutination inhibitiontest
d. Direct fluorescent antibodytest
Feedback: A direct fluorescent antibody test (DFA) can be used, for example, to detect antigens
of Legionella, Bordetella, Neisseria, and Treponema in appropriate clinical specimens. However
DFA can detect only antigens, not antibodies.
172. That portion of an enzyme which is separated from its cofactor is called a(n):
a. Partial enzyme
b. Isoenzyme
c. Coenzyme
d. Apoenzyme
Feedback: A partial enzyme is not an actual term used in the laboratory. An isoenzyme is a
related enzyme with a different chemical structure. Finally, a coenzyme is a non-protein
molecule (often a vitamin) that helps an enzyme become active.
173. Increases in the MB fraction of CK is associated with:
a. Liver disease
b. Bone disease
c. Muscle trauma
d. Mycardial infarction
Feedback: The isoenzyme CK-MB is fairly cardiac specific. It is elevated within 6 to 8 hours of a
myocardial infarction, and remains elevated for about 2 to 3 days post infarction.
174. A normal hemoglobin molecule is comprised of the following:
a. Ferrous iron and four globin chains
b. Four heme and four globin chains
c. Four heme and one globin chains
d. One heme and four globinchains
Feedback: A normal hemoglobin molecule consists of a tetramer with a molecular weight of
64,500daltons. The constituent monomers are each composed of one heme molecule, and one
polypeptide globin chain.
175. A delayed hemolytic transfusion reaction is most likely to be the result of which of the
following antibodies:
a. Lea
b. A
c. Jka
d. B
Feedback: Jka(Kidd) antibodies are very dangerous; they disappear from circulation quickly and
can cause severe delayed transfusion reactions.
176. Increased excretion of creatinine would be expected in which of the following groups:
a. Elderlymales/females
b. Adult males
c. Adult females
d. Children/infants
Feedback: Diets high in meats, and extreme exercise will cause a significant increase in
creatinine excretion.
177. Pheochromocytoma is a tumor of the adrenal medulla that results in elevated urinary
levels of all but which of the following:
a. Cortisol
b. Free catecholamines
c. Metanephrines
d. Vanillylmandelic acid
Feedback: Pheochromocytoma is an adrenal or extra adrenal neoplasm that secretes
catecholamines. Patients with pheochromocytoma often exhibit persistent and paroxysmal
hypertension. The single best screening test is urinary metanephrines. Urinary free
catecholamines, and vanillylmandelic acid are also elevated.
Question Excerpt

1. Which of the following media is used for culturing Salmonella?

A. VL-broth
B. Sabouraud agar
C. Slanetz Bartley
D. Selenite Broth

Explanation: VL-broth is used for anaerobes. Sabouraud agar is used for fungi. Slanetz-Bartley is used for
enterococci. Selenite broth is an example of a Selectively Multiplying liquid media used to culture
Salmonella.

2. Arabinose is a test-tube test used for:

A. Enterococci
B. Salmonella
C. G- bacteria
D. Neisseriae

Explanation: Turns yellow = positive, remains green = negative.

3. Choose the incorrect statement about Endo agar:

A. Only some G+ bacteria can grow on it selectively


B. Can be used to differentiate lactose fermentation
C. Is similar to McConkey medium
D. Combines selective and diagnostic properties

Explanation: Only some G- bacteria can grow on it selectively.

4. Choose the correct statement about Antigen detection:

A. It is an indirect method
B. Negative result means presence of microbe in the patient’s body
C. It is carried out in the laboratory using antibodies of animal origin
D. Uses a sample of patients saliva

Explanation: It is a direct method. Positive result means presence of the microbe in the patient‘s body.

5. The third phase of the PCR reaction involves:

A. Detection of amplification product


B. Amplification of DNA
C. Breakdown of DNA
D. Obtaining isolated DNA

Explanation: The amplification product should be detected by gel electrophoresis or ELISA in third
phase.

6. Chocolat agar is an example of a pure enriched medium used for culturing:


A. Streptococci
B. Hemophili
C. Enterococci
D. Fungi

Explanation: It is used for pathogenous Neisseriae and Hemophili (that do not grow even on blood agar)

7. When using Hanja medium, if the colour of the whole medium is yellow it means:

A. Formation of H2S
B. Bacterium is not a glucose fermenter
C. Bacterium is a lactose fermenter
D. Bacterium is not a lactose fermenter

Explanation: Formation of H2S causes bottom part of medium to turn black.

8. Optochin is used to test:

A. Lactose fermentation
B. Motility
C. Glucose fermentation
D. Antibiotic susceptibility

Explanation: for answers b) c) and d), Hanja medium + MIU is used.

9. Löwentein-Jensen medium is used for culturing:

A. Enterobacteria
B. Tuberculosis
C. Salmonella
D. G+ bacteria

Explanation: It is a green coloured solid enriched media in a test tube, used for TBC.

10. Arabinose is a test-tube test used for:

A. Salmonella
B. Neisseriae
C. Enterococci
D. G- bacteria

Explanation: Turns yellow = positive, remains green = negative.

11. Choose the incorrect statement about E-tests

A. The testing strip has a rising Atb concentration


B. The test is qualitative
C. The strip has a scale
D. Principally similar to diffusion disc test

Explanation: The E-test is quantitative


12. The MIC Value is:

A. the highest concentration that inhibits growth


B. the highest concentration that allows growth
C. the lowest concentration that inhibits growth
D. the lowest concentration that allows growth

Explanation: MIC value is used in the microdilution test.

13. In microdilution testing, if MIC Value > breakpoint;

A. strain is suceptible
B. strain is resistant
C. a different test is needed
D. none of the answers are correct

Explanation: MIC > breakpoint = strain is resistant

14. Haemagluttination Inhibition Test:

A. is not an agglutination reaction


B. is a neutralisation reaction
C. result with a dense round target is positive
D. all answers are correct

Explanation: Example of use: we can read HIT results for tick-born encephalitis.

15. Choose the correct statement about ASO:

A. is a direct diagnostics reaction


B. the aim is to find a pathogen
C. ASO examination is indicated in rheumatoid fever
D. all of the answers are correct

16. Which of the following is NOT considered a classification of PCR:

A. Specific PCR
B. Multiplex PCR
C. Universal PCR
D. Segmental PCR

17. In the first phase of PCR reaction:

A. We have to find isolated DNA


B. Product is initially detected
C. We have to amplify the DNA
D. None of the answers are correct

18. Catalase test is used for:

A. diagnostics of G+ cocci
B. diagnostics of G+ rods
C. uses hydrogen peroxide
D. all answers are correct

19. Indol test positivity is indicated by presence of:

A. a red ring
B. colour change to yellow
C. colour change to blue
D. increased turbidity

Explanation: Is used as part of MIU complex test tube test.

20. Octal codes are used in:

A. MIU test
B. Optochin test
C. Plate tests
D. Catalase test

Explanation: It is a mathematical binary system used in the plate tests., e.g. ENTEROtest16

21. The identification of bacteria by serologic tests is based on the presence of specific antigens. Which
of the following bacterial components is least likely to contain useful antigens?

a. Ribosomes
b. Cell wall
c. Capsule
d. Flagella

22. Each of the following statements concerning the Gram stain is correct except:

a. Escherichia coli stains pink because it has a thin peptidoglycan layer


b. Streptococcus pyogens stains blue because it has a thick peptidoglycan layer
c. Mycoplasma pneumoniae is not visible in the Gram's stain because it does not have a cell wall
d. Mycobaterium tuberculosis stains blue because it has a thick lipid layer

23. An outbreak of sepsis caused by Staphylococcus aureus has occurred in the newborn nursery. You
are called upon to investigate. According to your knowledge of the normal flora, what is the most likely
source of the organism?

a. Nose
b. Colon
c. Vagina
d. Throat

24. Each of the following organisms is an important cause of urinary tract infections except:

a. Klebsiella pneumoniae
b. Escherichia coli
c. Bacteriodes fragilis
d. Proteus mirabilis

25. A 30 year old woman has non bloody diarrhea for the past 14 hours. Which one of the following
organisms is least likely to cause this illness?

a. Streptococcus pyogens
b. Clostridium difficile
c. Shigella dysenteriae
d. Salmonella enteritidis

26. Each of the following agents is a recognized cause of diarrhea except

a. Clostridium perfringens
b. Vibrio cholerae
c. Enterococcus faecalis
d. Escherichia coli

27. Each of the following statements about the classification of streptococci is correct except

a. Pneumococci (Streptococcus pneumoniae) are alpha-hemolytic and can be serotyped on the


basis of their polysaccharide capsule
b. Enterococci are group D streptococci and can be classified by their ability to grow 6.5% sodium
chloride
c. Viridans streptococci are identified by Lancefield grouping, which is based on the C
carbohydrate in the cell wall
d. Although pneumococci and the viridans streptococci are alpha-hemolytic, they can be
differentiated by the bile solubility test and their susceptibility to optochin

28. Which of the following bacteria has the lowest 50% infective dose (ID50)?

a. Campylobacter jejuni
b. Salmonella typhi
c. Vibrio cholerae
d. Shigella sonnei

29. Which of the following disease is best diagnosed by serologic means?

a. Pulmonary tuberculosis
b. Gonorrhea
c. Actinomycosis
d. Q Fever

30. The cogulase test is used to differentiate

a. Staphylococcus epidermidis from Neisseria meningitidis


b. Staphylococcus aureus from Staphylococcus epidermidis
c. Streptococcus pyogens from Staphylococcus aureus
d. Streptococcus pyogens from Enterococcus faecalis
31. Which of the following bacteria is not a most common bacterial pathogen causing diarrhea?

a. Shigella spp
b. Salmonella spp
c. Enterococcus faecalis
d. Campylobacter
32. Which of the following bacteria is rarely associated with Urinary Tract Infections?
a. E.coli
b. Enterobacter spp
c. Proteus spp
d. Shigella spp
33. Which of the following statement regarding Campylobacter jejuni is not correct?
a. It is commonly cultured in antibiotic containing media
b. Incubation temperature is 42 Oc
c. It is cultured in an atmosphere containing 5% O2 and 10% CO2
d. It is a normal flora of intestine.

34. Which of the following bacteria is not most important causes of acute bacterial meningitis?

a. Neisseria menigitidis
b. Streptococcus pneumoniae
c. Haemophilus influenzae
d. Streptococcus pyogens

35. Which of the following Gram negative rod is not a blood borne bacterial pathogen?

a. Shigella spp
b. Escherichia Coli
c. Klebsiella pneumoniae
d. Pseudomonas aeruginosa

36. Throat culture is not useful to diagnose

a. Streptococcal sore throat


b. Diphtheria
c. Thrush
d. Pneumonia

37. Stool culture is primarily recommended when the complaint is

a. Bloody diarrhea (dysentery, enterocolitis)


b. Watery diarrhea
c. for both bloody and watery diarrhea
d. an indication of anaerobic infection

38. Which of the following bacteria causing sexually transmitted disease cannot be grown on artificial
media?

a. Neisseria gonorrhoeae
b. Chlamydia trachomatis
c. Treponema pallidum
d. Both Treponema pallidum and Chlamydia trachomatis

39. Cold agglutinin test is useful for the diagnosis of

a. Mycoplasma pneumoniae
b. H. influenzae
c. N. menigitidis
d. Cryptococcus neoformans

40. Which of the following test is not recommended for the diagnosis of Syphilis?

a. VDRL test
b. Culture
c. FTA-ABS test
d. MHA-TP test
Saudi Counsel Exams for Lab. Technician

Instruction: Select one best answer for the following multiple choice Questions:

1. The concentration of total bilirubin in the serum of normal adult ranges at.
a. 0.1– 1 mg/dl
b. 0.1– 1 ug/dl
c. 1– 10 mg/dl
d. 10 – 20 mg/dl
e. 100 -110 mg/dl
2. Laboratory tests of hepatic function include:
a) Serum bilirubin
b) Urine bilirubin
c) Excretory function test
d) All of the above
3. Marked elevations in serum amylase levels are seen in:
a) Bone disease
b) Acute pancreatitis
c) Salivary gland lesions
d) Liver disease
e) B and C
4. Glycogen, the storage form of carbohydrates in the body is predominantly found in.
a) Liver tissue
b) Muscle tissue
c) Liver and muscle
d) Bone
e) B and D
5. The hormone insulin helps to maintain constant plasma glucose concentration by:
a) Allowing entry of glucose from plasma in to muscle and fat tissue
b) Moving glucose out of cells into the plasma
c) Inhibiting glycolysis
d) All the above mechanisms
e) None of the above mechanisms
6. In case of uncontrolled sever diabetes mellitus, the following biochemical abnormalities appear:
a) Acidosis
b) Ketosis
c) Dehydration
d) Electrolyte imbalance
e) All of the above
7. Sever diabetes mellitus bring about which condition
a) Acidosis and Ketosis
b) None of the above
c) Dehydration
d) Electrolyte imbalance
e) All of the above
8. In chemical or latent type diabetes:
a) The fasting blood sugur level exceeds 150 mg/dl.
b) Glucose and ketone bodies are found in the urine.
c) The 2-hour postprandial serum glucose level exceeds 180 mg/dl
d) The glucose tolerance test result is normal
e) None of the above statements is correct
9. Which statement is true about chemical or latent diabetes:
a) FBS >150
b) Presence of acidosis and ketosis
c) 2HPP >180
d) OGTT is normal
e) Presence of sugar in urine
10. A technician working in the clinical chemistry lab must immediately report to his supervisor
aplasma glucose level equal to:
a) 80 mg/dl
b) 70 mg/dl
c) 60 mg/dl
d) 50 mg/dl
e) 120 mg/dl
11. Test of intestinal function includes: / A test used to assess intestinal function:
a) Lactose tolerance test
b) D-xylose absorption test
c) Measurement of serum carotenoids
d) Fecal fat analysis
e) None of the above
12. In spectrophotometeric determination, the amount of light of a specific wavelength absorbed by
acolored solution is:
a) Directly proportional to the intensity of the color
b) Directly proportional to the concentration of solute in the solution
c) Both A and B
d) None of the above
13. In spectrophotometeric, the amount of light absorbed is:
a) Directly proportional to the amount of light transmitted
b) Inversely proportional to the amount of solute
c) Directly proportional to the concentration of the analyte
d) Inversely proportional to the absorbance
e) Inversely proportional to the emitted color
14. The concentration of Sodium in normal serum is: / normal Sodium in adult
a) 136 – 145 mmol/l / 135 – 145 mmol/l
b) 136 – 145 mg/dl / 110 – 220 meq/l
c) 1.36 – 1.45 mmol/l /1.35 – 1.45 mmol/l
d) 1.36 – 1.45 mg/dl / 20 – 50 mg/dl
e) None of the above / None of the above
15. The best estimate of plasma osmolality is represented by:/ The Principal of body water
osmolalityis:
a) Plasma Na+ concentration / Na+ concentration
b) Plasma K+ Concentration / K+ concentration
c) Plasma HCO3 Concentration / HCO3- concentration
d) None of the above / CL- concentration
16. The most important buffer system present in blood is:
a) The hemoglobin buffer system
b) The phosphate buffer system
c) The bicarbonate/carbonic acid buffer system
d) The acetate/acetic acid buffer system
e) None of the above
17. Jaundice, a condition characterized by deposition of bilirubin in the sclera of the eyes and in
theskin, becomes evident when the bilirubin serum level reaches a concentration of:
a) 1 mg/dl
b) 0.1mg/dl
c) Exceeds 2.5 mg/dl
d) Exceeds 5 mg/dl
e) None of the above
18. In significant bacteriuria the cfu/ml urine is:
a) >10^5 cfu/ml
b) < 10^5 cfu/ml
c) >10^3 cfu/ml
d) < 10^3 cfu/ml
19. The test used to differentiate pathogenic staphylococci is:
a) Catalase
b) Oxidase
c) Coagulase
d) ASO test
20. Group A Streptococci are differentiated from other Beta-haemolytic streptococci by
disccontaining:
a) Neomycin
b) Optochin
c) Bacitracin
d) Novobiocin
21. For blood culture, the best anticoagulant is:
a) Citrate
b) Oxalate
c) Sodium polyanesthol sulphonate (SPS)
d) Heparin
22. The IMVIC reaction of E. coli are:
a) ++--
b) --++
c) +++-
d) ---+
23. On EMB or Mac Conkiy agar media, members of shigella-salmonella group produce
smallcolonies which are:
a) Black
b) Green
c) Pink
d) Colorless
e) Yellow-brown
24. The best specimen for recovery of polio virus for culture is:
a) C S F
b) Blood clot or throat washing
c) Buffy coat
d) Feces
e) None of the above
25. The diagnostic stage for Giardia lamblia in stool is:
a) Cyst
b) Trophozite
c) Cyst and/or trophozite
d) All of the above
e) None of the above
26. Man may acquire toxoplasma from:
a) Mice
b) Cats
c) Dogs
d) Pigs
27. All Leishmanial infections are transmitted by / Vector for Leismania infection
a) Tsetse fly/ Tsetse fly
b) Sand fly/ Sand fly
c) House fly/ House fly
d) Horse fly/ Anopheles mosquito
28. Dwarf tapeworm:
a) Tania saginata
b) Hymenolepis nana
c) Taenia solium
d) Dipylidium caninum
29. Viruses resemble living thing; they:
a) Are motile
b) Are crystalline
c) Grow
d) Reproduce
30. Which of the following culture media is suitable for doing semi-quantitative bacterial count of
urine specimens?
a) MacConkey’s agar
b) Blood agar
c) CLED medium
d) XLD medium
e) Mannitol salt agar
31. Quellung’s test is positive with the following organisms EXCEPT:
a) Streptococcus pneumonia
b) Haemophilus influenza
c) Klebseilla pneumonia
d) Corynebacterium diphtheria
32. Growth of vibrio cholera on TCBS medium produces:
a) Blue colonies
b) Yellow colonies
c) Green colonies
d) White colonies
33. The following organisms are lactose fermenters EXCEPT:
a) E1. Colli
b) Enterobacter cloacae
c) Shigella sonnei
d) Proteus spp.
34. All are true for salmonella typhi EXCEPT:
a) Human is the main reservoir or carrier.
b) Possesses H & O antigen
c) Oxidase positive
d) Produces H2S on TSI, XLD & HE media
e) Produces acid from glucose but no gas
35. All is true for salmonella EXCEPT
a) the main reservoir is human
b) Contains O and H antigen
c) Urease, KCN and phenylalanine negative
d) H2S positive with gas
e) Indole positive
36. The following are members of Enterobacteriaceae family EXCEP:
a) Klebsiella rhinoscleromatis
b) Serratia liquefaciens
c) Providencia alcalifaciens
d) Yersinia pseudotuberculosis
e) Campylobacter fetus
37. Which of the followings statements about Campylobacter Jejuni is false?
a) Gram negative curved bacilli
b) Arranged in pairs (see-gull)
c) Slow growth
d) Grows on XLD medium
38. Direct gram-staining is routinely performed on the following specimens EXCEPT:
a) Pus, aspirated fluids and discharges
b) CSF
c) C. sputum
d) Urine
e) Stool
39. When using a X10 eye piece and the X10 objective of a bright field microscope, the total
magnification is:
a) 10
b) 100
c) 1000
d) 10000
40. Which of the following microscopes would you use to examine a smear of bacteria stained
byGram stain?
a) Bright field microscope
b) Dark field microscope
c) Dissecting microscope
d) Fluorescent microscope
41. Which of the following microscopes would you use to examine a smear stained with a
fluorochrose:
a) Bright field microscope
b) Dark field microscope
c) Polarizing microscope
d) Fluorescent microscope
42. The resolution of the bright field microscope is:
a) 2.0 um
b) 0.2 um
c) 0.02 um
d) 0.002 um

43. Bacteriological culture media are generally sterilized using:

a) The autocalve
b) The oven
c) Filtration
d) Bunsen burner

44. Serum can be sterilized using membrane filter of porosity / Bacterial sterilization of serum can
beaccomplished by filtration using a porosity of

a) 0.22 um/ 0.22ù


b) 2.2 um/ 0.8ù
c) 6 um/ 0.60ù
d) 8 um/ 5.0ù

45. The temperature-time cycle when using the autoclave to sterilize is:

a) 100 C /30 min


b) 121 C /30 min
c) 90 C /30 min
d) 100 C /15 min

46. Which of the following materials would you sterilize using the oven?

a) Culture media
b) Serum
c) Sand
d) Distilled water

47. The primary stain of the gram stain is:

a) Crystal violet
b) Safranin
c) Malachite green
d) Iodine

48. Which of the following reagents is not used in the gram staining technique?

a) Crystal violet
b) Safranin
c) Malachite green
d) Iodine

49. Which color do gram negative bacteria appear?

a) Blue
b) Green
c) Yellow
d) Red

50. Which objective of the microscope requires the use of immersion oil between the slide and
theobjective?

a) X10
b) X20
c) X40
d) X100

51. The solidifying agent in bacteriological culture media is:

a) Wax
b) Resin
c) Agar
d) Starch

52. Agar is used as a solidifying agent in bacterial culture medium at a concentration:

a) 10% - 11%
b) 8% - 9%
c) 1% - 2%
d) 1% - 3%
53. Which of the following is not true?

a) Agar is inert
b) Agar is nutritious
c) Agar solidifies at 42 C
d) Agar melts at 100 C

54. Bacteria which has an optimum temperature for growth at 60 C are termed:

a) Mesophilic
b) Zoophilic
c) Thermophilic
d) Halophilic

55. What organism that char. Sattelitism around the Staph. Aureus on blood agar:

a) H. influenza
b) H. dueriyi
c) C. tetani
d) None of the above

56. Parasite that has a chronatiodal bodies or the cigar shaped:

a) E. Histolytica
b) E. coli
c) Endolimax nana
d) Balantidium Cole
e) E. hartmani

57. Ingestion of RBC in a trophozeite:

a) E. histolytica
b) E. coli
c) Endolinax nana

58. Sprozoite in man invades:/ Sproziote of malaria invades man through the:

a) WBC / White Blood cell


b) RBC / Red blood cell
c) Reticuloendothelial / RES
d) Muscle tissue
e) All of the above

59. Egyptians Hematuria:

a) fasciola Hepatiea
b) S. hematobium
c) S. japonicum
d) S. mansoni

60. The causative parasite of Egyptian bilharzias:


a) Trichomonas Vaginalis
b) Schistosoma haematobium
c) Schistosoma japonicum
d) Schistosoma mansoni
e) All of the above

61. Enlarge RBC: / Which malarial parasite cause enlargement of the RBC:

a) P. Vivax
b) P. malaria
c) P. falciparum
d) P. ovale

62. In urinary infection the ff. are free in urine:

a) Hematuria
b) Nitritis are positive
c) Leukocytes
d) All of the above

63. The probable glucose level:

a) below 25 – 30 mg/dl
b) 40 mg/dl
c) 60 mg/dl
d) Any of the above

64. In myocardial infection what enzyme are resp.

a) CK-MB
b) AST
c) ALT
d) CK – BB

65. In Antigen – Antibody reaction the panel should have such

a) 1 neg & 1 positive control


b) 2 neg & 1 positive control
c) 2 neg & 2 positive control
d) 3 neg & 3 positive control

66. Parasites that neither lay eggs nor:

a) T. trichiurea
b) Ascaris
c) Hookworm
d) Trichenilla

67. A parasite which does not deposit eggs in the intestine:

a) E. vermicularis
b) A. doudenale
c) A. lumbricoides
d) T. saginata
e) T. spiralis

68. Which of the following neither lays eggs nor deposits larvae in the intestinal canal:

a) Hook worm
b) Pin worm
c) Trichinella
d) Trichiuris

69. In blood transfusion the patients Red cells and donor plasma are req.

a) Minor cross matching


b) Major cross matching

70. What is the normal value of FBS if it is 100% venous blood?

a) 60-80 mg/dl
b) 100-120 mg/dl
c) 80-120 mg/dl
d) 200-210 mg/dl

71. In acute pancreatities what is the test to be determine:

a) Amylase
b) Lactate Dehydrogenises
c) AST
d) ALT

72. Other than Insulin what hormone that maintain the glucose level

a) Glucagon
b) Growth hormone
c) Somatostatin

73. Glomerular filtration Rate is indication of:

a) Measurement of Clearance of subs. That are freely filterable by the glomerular


b) Means of clearable of subs. That are freely filterable by the glomerular that secret bytubules and
is retainable
c) Any of above
d) All of the above

74. Increased Serum bilirubin:

a) Overloaded of Bilirubin in the liver


b) Dysfunction of hepatocellular
c) Esterification of conjugated
d) B&C
e) All of the above

75. Creatinine Determination:

a) Skeletal
b) Muscle
c) Heart
d) All of the above

76. The trophozoite which contains red blood cells in the cytoplasm belongs to this parasite:

a) Entamoeba hystolitica
b) Entamoeba gingivalis
c) Entamoeba coli
d) Iodamoeba butschlii
e) None of the above

77. What is the best time to collect blood for malarial smear:

a) At the beginning or just when the paroxysm is about to start


b) Period before the paroxysm start
c) Period when the paroxysm starts
d) Period after the paroxysm

78. The technician should inform the physician immediately when the blood glucose level is:

a) 60 mg/dl
b) 50 mg/dl
c) 80 mg/dl
d) 110 mg/dl
e) 120 mg/dl

79. Which of the following is an enrichment medium:

a) CLED
b) MAC
c) BAP
d) Chocolate Agar
e) Thiocynate broth

80. The only enrichment medium of following is:

a) Blood agar
b) Chocolate agar
c) Cystine lactose Electrolyte Deficient (CLED)
d) Selenite broth
e) Mac Conkey agar

81. Which of the following is an enriched medium:

a) Nutrient broth
b) Peptone water
c) Normal saline
d) Chocolate agar

82. Which of the following indicates a blood agar plate:

a) C. diphtheria
b) M. tuberculosis
c) B. pertussis
d) F. tularensis
e) None of the above

83. All are member of the Family Enterobacteriaccae are EXCEPT:

a) Klebsiella pneumonia
b) Yersinia pestis
c) Pseudomonas aeroginosa
d) Shigella dysenteriae
e) Campylobacter jejni

84. To obtain an isotonic media, the Sodium Chloride content should be:

a) 0.85 g %
b) 0.085 g%
c) 8.5 g%
d) 6.5 g%
e) 0.50 g%

85. Mycobacterium tuberculosis is called acid fast because:

a) It is easily stained by Grams stain


b) Once stained, it cannot be removed by acid
c) It easily decolorizes after stain
d) It is very easy to stain
e) It is stained by an alkaline stain

86. All of the following is a test for liver function except:

a) Bilirubin in urine
b) Bilirubin in Serum
c) Excretory function test
d) Enzyme test
e) All of the above

87. Most of the pathogenic stains of streptococcus belongs to group:

a) Viridan
b) Beta hemolytic
c) Alpha hemolytic
d) Group D
e) Micrococcaciae

88. Beta Hemolysis is enhanced when Group B Streptococcus in streaked at an angle in a blood
agarplate with:

a) Streptococcus pneumonia
b) Staphylococcus aureus
c) Micrococcus
d) Staphylococcus epidermidids
e) C. diphtheria

89. All statements are true about Proteus mirabilis and Proteus vulgaris except:

a) Oxidase negative and liquefies gelatin


b) Exhibits swarming on BAP and Mackonkey agar
c) Ureas positive
d) KCN and H2S positive
e) Positive to Indole test

90. All are true about Enterococci except:

a) They are Gamma hemolytic on BAP


b) Ferments Mannitol and Sorbitol
c) Reduce litmus milk with acid and coagulation
d) Growth in Methyline Blue medium
e) Inhabited growth with 6.5 % NaCl and at temperature 60 C

91. All are true about Enterococci except:

a) Have carbohydrate antigen or group D streptococci


b) Positive esculine-hydrolysis and (+) pyr test
c) Grow in the presence of bile salts
d) Do not grow in presence of 6.5 % NaCl nor at 45 C

92. Blood culture may be indicated for:

a) Clostridium tetani
b) Corynebacterium diphtheria
c) Shigella dysinteriae
d) Mycobacterium tuberculosis
e) None of the above

93. Streptococci responsible for the majority of human – infection are:

a) Group A
b) Beta – haemolytic
c) Bacitracin-S and CAMP (-)
d) All of these
e) None of these
94. The gonococcus and menigococcus can best be differentiated by their:

a) Fermentation of dextrose and maltose


b) Fermentation of maltose and lactose
c) Fermentation of maltose and sucrose

95. Identified by finding mainly the rhabditiform larvae in fresh stool:

a) Ascaris lumbricoides
b) Enterobiu vermicularis
c) Strongyloides stercoralis
d) Trichinella spiralis

96. The determination of ASO titer is used in the laboratory diagnosis of:

a) Toxic shock syndrome


b) Scarlet fever
c) Streptococci pharyngitis and cellulites
d) Acute glomerulonephritis
e) Rheumatic fever

97. Significant bacteriuria, pyuria, and alkaline urine probably indicate urinary infection due to:

a) Staphylococcus epedermidis
b) E. coli
c) Proteus vulgaris
d) Pseudomonas aeruginosa
e) Enterococcus faecalis

98. Which of the following are prokaryotes?

a) Bacteria
b) Rickettesia
c) Fungi
d) Protozoa
e) Chlamydia

99. The organs of adhesion in bacteria are:

a) Flagella
b) Fimbriae
c) Microcapsule
d) Slim layer

100. The thickness of bacterial cell wall ranges between

a) 2-5 nm
b) 5-10 nm
c) 10-15 nm
d) 15-20 nm
101. Which contain more peptidoglycan

a) Gram positive bacilli


b) Gram negative bacilli
c) Chlamydia
d) Rickettsiae

102. Endotoxins are chemically:

a) Mucopeptides
b) Proteins
c) Lipopolysacharides
d) Polysacharides

103. Minimum time required for sterilization by moist heat at 136 C is

a) 2 hours
b) 1 hour
c) 45 minutes
d) 15 minutes
e) 3 minutes

104. Which of the following is biological indicator used to monitor steam sterilizers.

a) Pseudomonas aeruginasa
b) Bacillus stearothermophilus
c) Bacillus anthraces
d) Clostridium difficile

105. Bacterial genes are transferred by means of all following except:

a) Transformation
b) Transduction
c) Mutation
d) Conjugation

106. The rate of decolonization is the fastest in Grams stain with

a) Acetone
b) Alcohol
c) Mixture of Acetone and Alcohol
d) Acetone plus iodine

107. Best method of staining bacterial capsule is:

a) India ink preparation


b) Grams stain
c) Leishman’s stain
d) Malachite green

108. Which of the following is not true about Nutrient agar:


a) It melts at about 95 C
b) It solidifies at about 34 C
c) It doesn’t inhibit the growth of bacteria
d) It adds nutritive value to the medium

109. All of the following are transport media except:

a) Ammies medium
b) Stuart’s medium
c) Carry Blair’s medium
d) Alkaline peptone water

110. The usual concentration of blood in the blood agar medium is:

a) 2%
b) 5%
c) 10%
d) 15%

111. The sources of Peptone include:

a) Heart muscle
b) Casein
c) Fibrin
d) Soya four
e) All of the above

112. The indicator in Mac Conkey agar is:

a) Neutral Red
b) Methyl Red
c) Phenol Red
d) Bromophenol Blue

113. For providing micro aerophilic atmosphere the preferred mixture is:

a) Nitrogen + Carbon dioxide


b) Hydrogen + Carbon dioxide
c) Oxygen + Carbon dioxide
d) None of the Above

114. All of the following are oxidase positive except:

a) Neisseria gonorrhea
b) Pseudomonas aeruginosa
c) Salmonella typhi
d) Vibrio cholera

Brucella

115. Chemically the oxidase reagent is:


a) Hydrogen peroxide
b) Tetra methyl-P-phenylene – diamine
c) Tributyrim
d) None of the above

116. Which of the following produce urease:

a) Escherichia Coli
b) Proteus Vulgaris
c) Salmonella gallinarum
d) Streptococcus haemoliti

117. Selective medium used for the isolation of Staphylococcus aureus is:

a) 2% milk agar
b) 10% Sodium chloride broth
c) Thioglycollate broth

118. The most commonly tested virulence factor in staphylococci is:

a) Coagulase
b) Hyaluronidase
c) Fibrinolysin
d) Enterotoxin

119. Group B streptococci cause:

a) Neonatal meningitis
b) Endocarditis
c) Septic abortions
d) All above

120. All following species of Staphylococcus do not produce coagulase except

a) S. Capitis
b) S. hominis
c) S. intermmedius
d) S. haemolyticus

121. Haemophilus influenza cause:

a) Meningitis
b) Respiratory infections
c) Conjunctivitis
d) Otitis media
e) All of the above

122. All following produce oxidase except:

a) Haemophilus species
b) Gardnerella vaginalis
c) Eikenella corrodens
d) Chromobacterium species

123. Which of the following can grow on MacConkey agar:

a) Chromobaterium violaceum
b) Haemophilus parainfluenza
c) Gardnerella vaginalis
d) Eikenella corrodens

124. The colonies of Bordetella pertussis look like:

a) Proached egg
b) Mercury drops
c) Sliver paints
d) Draughtsman

125. The best method of sample collection for Bordetella pertussis:

a) Pernasal swab
b) Cough plate method
c) Post nasal swab
d) Throat swab

126. The best medium for the isolation of Bordetella pertussis is:

a) Chocolate agar
b) Bordet—Gangou medium
c) Diamidine-penicillin-flouride medium
d) Chocolate agar with bacitracin

127. Which of following is capable of growth at 22 C

a) Neisseria meningitides
b) Neisseria lactamica
c) Neisseria catarrhalis
d) Neisseria gonorrhoeae

128. Neisseria meningitides utilizes:

a) Glucose and maltose


b) Maltose and sucrose
c) Sucrose and lactose
d) Glucose and lactose

129. Which of the following are true about Neisseria meningitides:

a) These are Gram negative diplococcic


b) Oxidase positive
c) Require 5-10% of carbon dioxide for growth
d) All above
130. The chemotherapeutic agents contained in modified New York City (MNYC) medium are:

a) Amphotericin B
b) Linocomycin
c) Colistin
d) Trimethroprim
e) All of above

131. Which of the following are typical about morphology of Corynebacterium diphtheria:

a) Gram positive thin bacilli


b) Presence of metachromatic granule
c) Club shaped at the ends
d) Arranged at acute angles
e) All above

132. Morphological features of C. diphtheria are best developed on:

a) Blood agar
b) Blood tellurite agar
c) Loeffler serum slope
d) Chocolate agar

133. JK group of corynebacteria are universal sensitivity to:

a) Teracycline
b) Vancomycin
c) Erythromycin
d) All above

134. Which test is used to ascertain toxigenity of C. diphtheria:

a) Elek’s test
b) Dick test
c) Schick test
d) None of the above

135. A smear stained with Ziehl Neelsen stain should not be declared negative unless it has been
examined for at least:

a) 5 minutes
b) 10 minutes
c) 15 minutes
d) 20 minutes

136. In Ziehl Neelsen staining used for M. leprae, the decolorizing agent used is:

a) 5% sulphuric acid
b) 10% sulphuric acid
c) 15% sulphuric acid
d) 20% sulphuric acid

137. Voges proskauer (VP) negative organism is:

a) Shigella
b) Klebsiella
c) Enterobacter
d) Serratia

138. Salmonella ‘O’ antigens are also share with:

a) Citrobacter
b) Escherichia coli
c) Shigella
d) Proteus
e) All above

139. Enrichment medium for Salmonella is:

a) Tetrathionate broth
b) Selenite F
c) Strontium chloride broth
d) Any of above

140. Determination of antibodies to Vi antigen in Widal test helps in diagnosis of:

a) Early stage of typhoid fever


b) Typhoid encephalopathy
c) Carrier state
d) Virulent form of typhoid fever

141. ONPG test is positive in all the strains of:

a) S. Flexneri
b) S. boydii
c) S. sonnei
d) S. dysenteriae

142. IgM is

a) Acute Immunoglobulin
b) Immunoglobulin in allergic condition
c) Chronic Immunoglobulin
d) Alpha globulin

143. Intramuscular injection increase the activity of the following enzyme:

a) GPT
b) CPK
c) CK-MB
d) LDH
144. In blood Glucose estimation the following is required

a) Fasting blood glucose


b) Glycosylated Hb
c) PPBS
d) Fructoseamine
e) All of the above

145. Female anopheles transmits which disease:

a) Lishmanisasis
b) Filariasis
c) Trypanosoma
d) Malaria

146. Na, K and Cl are best measure?

a) Spectrophotometer
b) Flame photometer
c) ELISA
d) RIA

147. Glucose -6- phosphate dehydrogenase is:

a) Cardiac enzyme
b) Pancreatic enzyme
c) AMP pathway enzyme
d) Prostatic enzyme

148. Estimation of G6PDH is a test

a) To diagnose hemolytic anemia


b) To diagnose myocardial infection
c) T diagnose pancreatic disease

149. Cardiac LDH can be substituted by one of the following

a) GOT
b) GPT
c) CPK
d) HBDH

150. One of the following is specific diagnostic enzyme

a) GGT
b) GOT
c) LDH
d) CPK

151. Alkaline Phosphatase is


a) Live enzyme
b) Placental enzyme
c) Bone enzyme
d) All of the above
e) None of the above

152. One of the following is pancreatic enzyme:

a) CPK
b) Lipase
c) GOTD
d) Acid phosphates

153. Vaginal discharge occur with all except:

a) Gardenela
b) Chlamydia
c) Candida
d) Trichomonous Vaginallis
e) Gonorrhea

154. Trypnosoma move by

a) Pseudopodia
b) Cilia
c) Flagella
d) None of the above

155. Giardia move by

a) Pseudopadia
b) Cilia
c) Flagella
d) None of the above

156. Entamoeba histolytica cyst is appear in which part of gastrointestinal tract:

a) Stomach
b) Small intestine
c) Large intestine
d) None of the above

157. The following parasite can seen in the blood:

a) Malaria
b) Filaria
c) Trypanosoma
d) Lishmania
e) All of the above
158. One of the following is Gram positive bacilli:

a) Anthrax
b) Niesseriae
c) E. coli
d) None of the above

159. Sensitivity and specificity are:

a) Directly related
b) Reversely related
c) They mean the same
d) None of the above

160. A dichromatic analysis is carried to increase

a) Specificity
b) Sensitivity
c) Linearity
d) None of the above

161. Serious orgasm that cause food poisoning:

a) Staph. Albus
b) Salmonella entreritidis
c) Clostridium spelica
d) Salmonella typhi
SAUDI COUNCIL EXAMINATION
Choose the correct answer:

1. The best anticoagulant in blood culture is?


a. sodium oxalate
b. sodium citrates
c. sodium sulphate
2. All the following samples are stained using gram stain except?
a. water culture
b. sputum
c. stool

3. The percentage of sodium chloride in agar culture is?

a. 0.058%
b. 58.0%
c. 0.58%

4. Agar is characterized by all the following except?

a. freezing point is 42c & melting point is 100c


b. nutritive
c. un-nutritive

5. The normal value of (Na) in serum is?

a. 135-145 mg/dl
b. 135-145 mmol/dl
c. 135-145 mmol/l

6. Gram stain starts with?

a. Iodine
b. crystal green
c. methyl violet

7. The causative of toxoplasma is?

a. Flees
b. Dogs
c. Cats

8. Gram negative samples are stained with?

a. yellow color
b. blue color
c. red color

9. The color of cholera when cultured on TCBS?


a. Blue
b. Green
c. Yellow

10. The diameter of micro pore filter used in bacteria filtration is?

a. 0.033microns
b. 2.2microns
c. 0.22microns

11. Leishmania is transported by?

a. Mosquitoes
b. Pugs
c. sand fly

12. To investigate one sample containing floro-carbon we use which microscope?

a. light microscope
b. brilliant microscope
c. florescent microscope

13. Virus is similar to live organisms in?

a. Growth
b. Movement
c. Reproduction

14. The largest resolution of the microscope is?

a. 0.02microns
b. 2.0microns
c. 0.2microns

15. Which of the following is liver function test?

a. liver enzymes
b. serum + urine billrubin
c. all of the previous

16. Which of the following causing enlargement of RBCs?

a. Reticulocytes
b. Leishmania
c. Plasmodium

17. The best sample for the culture of children paralysis virus is?

a. anal swab
b. blood culture
c. stool culture
18. Dry air oven is used to sterilize?

a. Water
b. culture media
c. sand

19. The diabetic patient is going to comma when blood glucose is?

a. 120 mg/dl
b. 160 mg/dl
c. less than 50mg/dl

20. To differentiate between the two streptococci group which enzyme is used?

a. Coagulase
b. Kinase
c. Catalase

21. The microscope which used in investigate syphilis is?

a. light microscope
b. ultra-violet microscope
c. dark field microscope

22. The antibiotic disc which differentiate between group A streptococci and other groups is?

a. Neomycin
b. Tetracycline
c. Bacitracin

23. All of the following is true for salmonella except?

a. Motile
b. produces H2S
c. oxidase positive

24. Amylase value is high in the following disease?

a. salivary glands
b. pancreas diseases
c. all of the previous

25. One jaundice patient has yellow skin, his billrubin is

a. 2.5mg/dl
b. 1.2mg/dl
c. 5.0mg/dl

26. Acute diabetic patient has?

a. blood sugar more than 150mg/dl


b. blood sugar more than 180mg/dl
c. glucose & acetone in urine

27. The normal value of billrubin is?

a. 0.2-2.0mg/dl
b. less than 2.0mg/dl
c. less than 1.0mg/dl

28. Diabetic patient has one of the following symptoms?

a. Acidosis
b. Alkalosis
c. Dryness

29. Insulin is regulating blood sugar by?

a. increase the influx of glucose into cells


b. activate glycogenesis
c. all the previous

30. The nutritive substance in culture media is?

a. Sugar
b. Starch
c. Agar

31. The best media for urine culture is?

a. blood agar
b. chocolate agar
c. cled agar

32. The sterilization of autoclave is?

a. 85c for 30min


b. 150c for 30min
c. 121c for 15min

33. Shigella soni colored in maconkey & EMB?

a. Colorless
b. Red
c. pink

34. The oil emersion lens is

a. 10
b. 40
c. 100

35. The dwarf worm is?


a. Schistosoma
b. tenia saginata
c. H.nana

36. All of the following are present in gram stain except?

a. Safranine
b. Iodine
c. malachite green

37. The diagnostic stage of amoeba is?

a. Cyst
b. Trophozoite
c. all previous

38. To check the intestinal efficacy the following test is done?

a. Pepsin
b. Lipase
c. stool fats

39. The worm that causes blood in urine?

a. Ascaris
b. Fasciola
c. schistosoma haematobium

40. The significant count of bacteria to be inflammation is?

a. less than 10^5


b. more than 10^3
c. more than 10^5

41. To investigate gram stained samples we use the following microscope?

a. double phase microscope


b. florescent microscope
c. light microscope

42. The most important buffer system in blood is?

a. acetic acid system


b. hemoglobin system
c. bicarbonates system

43. All the following bacteria are interobacter except?

a. E.coli
b. Proteus
c. non of the previous
44. In the microscope when using objective lens 10 & eye lens 10 thee magnification power of the
microscope is?

a. 10
b. 1000
c. 100

45. All the following parameters affecting gram staining except?

a. use H2SO4
b. add absolute alcohol after washing
c. delaying the dryness of the slide

46. Glycogen is stored in?

a. Liver
b. Muscles
c. all the previous

47. Malaria shizonts are present in?

a. reticulo-endothelial
b. leukocyte
c. RBCs

48. Comma of diabetic patient shows?

a. glucose greater than 200mg/dl


b. glucose less than 200mg/dl
c. glucose greater than 500mg/dl

49. Blood donor selection in KSA are all of the following except?

a. Hb% 12.5-16
b. free from syphilis
c. one year after delivery

50. Solidifying agent in culture media is?

a. Wax
b. Starch
c. Agar

51. Agar which used as a solidifying agent in culture media concentration is?

a. 5-9%
b. 4-9%
c. 1-2%

52. To sterilize culture media we use?

a. Boiling
b. hot oven
c. autoclave

53. Bone matrix can also called?

a. vascular tissue
b. fibrous tissue
c. osteon

54. Unidirectional movement of WBCs directly to its target is?

a. Sliding
b. Phagocytes
c. chemo taxis

55. When focusing a stained smear under oil immersion field the magnification is?

a. 10
b. 40
c. 100

56. In myocardial infection?

a. level of LHD high


b. level of GOT high
c. level of CK-MB high

57. Variation in RBCs size?

a. Microcytosis
b. Macrocytosis
c. anisocytosis

58. Pretranfusion test that is performed using the patient red cells and donor plasma?

a. Anti body screen


b. major x-matching
c. minor x-matching

59. Serum LDH is elevated in all the following except?

a. skeletal disease
b. cardiac/ hepatic diseases
c. renal disease

60. Elevated sodium &chloride is seen in?

a. shock
b. diabetic acidosis
c. severe dehydration

61. Which test is better to diagnose chronic bile duct obstruction?


a. total bilirubin
b. S-GOT
c. ALP

62. Generally diagnosed by recovery & identification of typical larva in stool?

a. hook worms
b. t.trichura
c. s.stercoralis

63. Enlarged RBCs are common in?

a. p.malaria
b. p.falciparum
c. p.vivax

64. Which of the following is not laying eggs in small intestine?

a. hook worm
b. t.saginata
c. pin worm

65. Blood sample is used to diagnose?

a. C.tetani
b. C.diphteria
c. non from them

66. How much water should we add to 500ml of a solution of 10% of NAOH to bring it to 7.5%?

a. 666
b. 250
c. 166

67. While using the pregnancy test we are measuring?

a. total HCG
b. beta HCG &LH
c. beta HCG

68. With age the renal threshold for glucose?

a. Increase
b. Decrease
c. does not change

69. calibrator sera are?

a. secondary standards
b. internal standards
c. primary standards
70. a buffer made of?

a. a strong acid + a strong salt


b. a weak acid + a weak salt
c. a weak acid + a strong salt

71. A standard micro plates in ELISA test has?

a. 98 wells
b. 94 wells
c. 96 wells

72. The enzyme in ELISA testing is present in the?

a. Buffer
b. micro plate
c. conjugate

73. Antigen antibody complex are?

a. weakly bound
b. no bounds
c. strongly bound

74. Washing must be done in all heterogeneous ELISA technique because?

a. increase the specificity


b. increase the sensitivity
c. it removes the excess binding

75. The label in ELISA tests is?

a. radio active substance


b. antibody
c. enzyme

76. The difference between plasma & serum is that plasma?

a. does not contain fibrinogen


b. has more water
c. contains fibrinogen

77. Five ml of colored solution has an absorbance of .500nmThe absorbance of 10ml of the same
solution is?

a. 1.000nm
b. 0.250nm
c. 0.500nm

78. Plasma or serum should be separated at the earliesttime for estimation of glucose because?

a. glucose value increases with time


b. lyses of blood will occur
c. glucose value decreases with time

79. Sensitivity & specify are?

a. directly related
b. they mean the same
c. none of above

80. A dichromatic analysis is carried to increase?

a. Linearity
b. Specify
c. Sensitivity

81. Causes of high serum bilirubin are?

a. overload on liver
b. haemolysis
c. all of the previous

82. Polio myeletis is transmitted through?

a. Skin
b. Respiration
c. feco-oral

83. Malaria does not grow in?

a. EDTA blood
b. heparin zed blood
c. plasma

84. Serious that causes food poisoning?

a. staph albus
b. salmonella typhi
c. salmonella enteritidis

85. Hemophilia man married to normal woman the incidence of his children is?

a. carrier male
b. diseased female
c. carrier female

86. Which of the following causes UTI & INDOL positive?

a. Klebsiella
b. Staphylococci
c. E.coli

87. Blood transfusion can transmit?


a. HIV
b. CMV
c. all of the previous

88. ADH is secreted from?

a. thyroid gland
b. anterior pituitary
c. posterior pituitary

89. One of the following enzymes is effected by hemolysis?

a. SGOT
b. SGPT
c. LDH

90. HbA1C of diabetic patient is important for?

a. he has to come fasting


b. short term follow up
c. long term follow up

91. For glucose tolerance test?

a. collect 5 blood samples only


b. collect 5 urine samples only
c. collect 5 blood samples + 5 urin samples

92. One of the following heart enzymes is measured after 4-8hr of chest pain?

a. GOT
b. LDH
c. CPK

93. Light effects one of the following?

a. Glucose
b. Urea
c. Billrubin

94. One of the following is specific diagnostic liver enzyme?

a. GOT
b. LDH
c. GPT

95. One of the following is important before anesthesia?

a. alkaline phosphates
b. acidic phosphates
c. pseudocholine esterase
96. Acid phosphates is?

a. heart enzyme
b. liver enzyme
c. prostatic enzyme

97. In uric acid estimation?

a. its affected by carbohydrate meal


b. no need for fasting
c. the patient has to come fasting

98. All of the following are affected by meal except?

a. Glucose
b. Albumin
c. Creatinine

99. Na is the main?

a. intra cellular anion


b. intra cellular cation
c. extra cellular cation

100. One of the following electrolytes is effected by hemolisis?

a. Ca
b. Mg
c. k

101. The best kidney function test?

a. Urea
b. total protein
c. creatinine clearance

102. For GGT in adults the dose of glucose is?

a. 50gram
b. 100gram
c. 75gram

103. For GGT in children the dose of glucose is?

a. 30mg
b. 15gram
c. 30gram

104. Random blood glucose gives an idea?

a. to justify blood glucose


b. fasting patient
c. blood glucose in urgent cases

105. Immunoglobulin is?

a. B globulin
b. Alfa globulin
c. Gamma globulin

106. Exogenous triglecride is carried on?

a. VLDL
b. HDL
c. Chylomicron

107. Endogenous triglecride is carried on?

a. LDL
b. HDL
c. VLDL

108. Harmful cholesterol is carried on?

a. HDL
b. VLDL
c. LDL

109. Useful cholesterol is carried on?

a. Chylomicron
b. LDL
c. HDL

110. For lipid investigation patient has to fast?

a. 4-6hr
b. 6-8hr
c. 12-14hr

111. To measure G6PD we use?

a. Plasma
b. Serum
c. whole blood on EDTA

112. Amoeba moves by?

a. Cilia
b. Flagella
c. Pseudopodia

113. One of the following is capsulated bacteria?


a. Streptococci
b. E.coli
c. klebsiella
GENERAL QUESTION

1. One is not found (involve) in colorimetric:


a) Cuvett
b) light source
c) Photo sensor and analyzer
d) fuel source
e) Filter
2. The atomic mass of an atom is the number of:
a) Protons and neutrons
b) Protons only
c) Neutrons only
d) Electrons
e) None of the above
3. Deionization of impure water means:
a) Boiling
b) Filtration
c) Exchange of protons and electrons
4. If you see this sign (symbol) in the lab it means:

a) Flammable
b) Corrosive
c) Oxidizing
d) Explosive
e) Toxic
5. Most common method (technique) used to detect hormone amount in the laboratories:
a) Spectrophotometry
b) Enzyme Linked Immuno Surbant Assay (ELIZA)
6. Test should be kept away from light: Serum bilirubin
7. Buffer solution is:
a) Change color when change pH
b) Resist acidic pH
c) Resist alkaline pH
d) Weak acid + weak base
e) Strong acid + weak base
8. An indicator is:
a) Change color with change pH
b) Resist acidic pH
c) Resist alkaline pH
d) Weak acid + weak base
e) Strong acid + weak base
9. Heparin is:
a) Protein
b) Enzyme
c) Polysaccharide
d) Oligosaccharide
e) None of the above
10. Not find in the flame photometry :
a) Burner
b) Filter
c) Fuel source
d) Cuvete for sample
e) Photo
11. Low effective sterilization with:
a) Ethyl alcohol 70%
b) Methyl alcohol
c) Chloroform
d) 5%phenol
e) 5% cresol
12. Prolonged fast cause:
a) Hyperglycemia
b) Keton in urine
13. Lens near the slid in light microscope :
ObjectiveEye lenses
14. Do not sterile with hot air oven:
a) Dry glassware
b) Oil
c) Powder
d) Waxes
e) Rubber gloves
15. Autoclave sterilize in temp: 121oC-20-30min
16. Alkali skin burn treatment by neutralization with:
a) Sodium bicarbonate powder
b) Boric acid
c) Acetic acid 1%
d) Cold water
e) Hot water
17. You do not take this stage for treatment of small cut In emergency :
a) Clean with soap and water
b) Do pressure with piece
c) Immediately rinse mouth well with water
d) Cover it with water dressing
e) Sterilize
18. the (u) unit used to evaluation of :
a) Protein in serum
b) Hemoglobin
c) Hormone assay
d) Enzymatic activity
19. During reaction of two chemical substances , the color produced is assessed by
a) Spectrophotometer
b) Flame photometer
20. To detect pH of solution use :
a) pH meter
b) Indicator
c) Micrometer
21. This Symbol means, when you see in laboratory :

a) acute flammable
b) Toxic
c) Corrosive
d) Explosive
22. The microscope which is used in investigate syphilis is?
a) light microscope
b) ultra-violet microscope
c) dark field microscope
23. How much water should we add to 500ml of a solution of 10% of NAOH to bring it to 7.5%?
a) 666
b) 250
c) 166
24. Calibrator sera are?
a) secondary standards
b) internal standards
c) primary standards
25. A buffer made of?
a) a strong acid + a strong salt
b) a weak acid + a weak salt
c) a weak acid + a strong salt
26. The difference between plasma & serum is that plasma?
a) does not contain fibrinogen
b) has more water
c) contains fibrinogen (Plasma)
27. Five ml of colored solution has an absorbance of .500nm the absorbance of 10ml of the same
solution is?
a) 1.000nm
b) 0.250nm
c) 0.500nm
28. Plasma or serum should be separated at the earliest time for estimation of glucose because?
a) glucose value increases with time
b) lyses of blood will occur
c) glucose value decreases with time
29. Wave Length Visible To Naked Eye:400-700 nm.
30. Purpose of standard deviation :
a) to measure external quality control
b) to measure internal quality control
c) precise & accuracy
d) both a & b
31. Substance used in catalyse reaction : H2O2

HAEMATOLOGY

1. Neutrophil count is high in:


a) Acute bacterial infection
b) Iron deficiency anemia
c) Megaloblastic anemia
2. HbA2 is consisting of:
a) 3 ά chains and 2 γ chains
b) 2 ά chains and 2 β chains
c) 2 ά chains and 2 δ chains
d) 2 ά chains and 3 δ chains
e) 3 ά chains and 2 δ chains
3. The main (most) Hb found in adult is:
a) Hb A
b) Hb A2
c) Hb F
4. Detection of malaria parasite is by: Thick blood film
5. Leukocyte that involve in Adaptive and Acquired immunity:
a) Lymphocyte
b) Neutrophil
c) Monocyte
d) Basophile
e) Eosinophil
6. Normal range of leukocyte is: 4-11 X 10^9
7. Reticulocyte is immature:
a) RBC
b) WBC
c) Platelet
8. Leukocyte responsible for cellular immunity:
a) T-lymphocyte
b) B- lymphocyte
c) Monocyte
d) Basophile
e) Eosinophil
9. Malaria infection transmitted by:
a) Male anopheles mosquito
b) Female anopheles mosquito
10. prothrombin time is used to detect (diagnose) disorders in:
a) Extrinsic coagulation pathway (factors)
b) Intrinsic coagulation pathway (factors)
11. Thrombin time is:
a) Extrinsic coagulation pathway (factors)
b) Intrinsic coagulation pathway (factors)
c) The time for conversion of prothrombin to fibrinogen in addition of thrombin
12. Most sever (serious) malaria infection caused by:
a) Plasmodium Falciparum
b) Plasmodium Malaria
c) Plasmodium Ovale
d) Plasmodium Vivax
e) All of the above
13. Leukocyte responsible for response to parasitic and allergic infection:
a) Lymphocyte
b) Nutrophile
c) Monocyte
d) Basophile
e) Eosinophil
14. With Romanowsky stain ,2-5 lobes and give violet or pinkish granules:
a) Neutrophile
b) Eosinophile
c) Basophile
d) Monocyte
15. Lymphocyte is elevated in :
a) Viral infection
b) Acute bacterial infection
c) Iron deficiency anemia
d) Megaloblastic anemia
e) None of the above
16. Thrombin time measure: Convert fibrinogen to fibrin with activate of thrombin
17. In presence of clotting defect one of this not measure: Leukocyte count
18. Bleeding time test detect the abnormality in : Defect in vessels and platelets
19. S hemoglobin is the same defect of thalassemia but the difference is :
a) Long B chain
b) Glutamic acid on B chain is substituted with valine
c) Glutamic acid on a chain is substitutes with valine
20. Hormone that cause replication of RBC: Erythropoietin
21. Which of White blood cell give immunoglobulin :
a) Lymphocyte
b) Neutrophil
c) Basophile
d) Monocyte
e) Eosinophil
22. Neutrophil is a common White blood cell present in blood and the percentage of presence is :
a) 90%
b) 15%
c) 75%
23. Normal range of erythrocyte: 4.5- 6.5X 10^9
24. One of these cell the largest leukocyte cell:
a) Neutrophil
b) Basophil
c) Monocyte
d) Lymphocyte
e) Eosinophil
25. Malaria schizonts are present in?
a) reticulo-endothelial
b) leukocyte
c) RBCs
26. Unidirectional movement of WBCs directly to its target is?
a) Sliding
b) Phagocytes
c) chemo taxis
27. Malaria does not grow in?
a) EDTA blood
b) heparinized blood
c) Plasma
28. Hemophilia man married to normal woman the incidence of his children is?
a) carrier male
b) diseased female
c) carrier female
29. IVY method of bleeding time: For vascular and platelets abnormalities (function)
30. Neutrophil: Most abundant in WBCs
31. HbA(Adult haemoglobin ) for adult, dominant in adult
32. HbF(Infant heamoglobin) fetal haemoglobin
33. Neutrophil: Firstly increased in bacterial infections
34. Prolonged application of tourniquet leads to
Venous stasis which result in increased calcium level
35. Best time for collection of blood for malaria :-
a) before and after paroxysm
b) shortly after paroxysm
c) later paroxysm
d) just before paroxysm
36. In folic acid deficiency what happens to rbcs :-
a) Enlarged RBCs ( Megaloblastic anemia ) = Macrocytic
b) Crenated RBCs
c) Haemolyzed RBCs
37. In iron deficiency anaemia :RBCs are smaller than normal = Microcytic
38. The malarial sporozoite in man invades:
a) white blood cells
b) RBCs
c) Reticulo-endothelial cells
d) all of the above
39. The defect of thalassemia occurs in:
a) Cycle of heme
b) Glubin chain
40. All Cells Are Nucleated EXCEPT:
a) LYMPH
b) MONOCYTES
c) RBCS
d) NEUTROPHIL
41. Romanowsky stain consists of:
a) Eosin + Alkaline methylene blue
b) Eosin only
c) Methylene blue only
d) Indian Ink

BIO-CHEMISTRY

1. One is not correctly paired:


a) α- cells → glucagon
b) β- cells → insulin
c) Parathyroid → calcium
d) Corpus luteum → Progesterone
e) Estrogen → seminiferous tubules
2. In case of obstructive jaundice one is commonly not found:
a) Total bilirubin is elevated
b) High bilirubin in urine
c) Direct bilirubin is within the normal
d) Dark color of urine
e) Clay color of stool
3. In case of Hemolytic jaundice one is not likely to be found:
a) Total bilirubin is elevated
b) High bilirubin in urine
c) No change in color of urine
d) Direct bilirubin is within the normal
e) Normal color of stool
4. One is not true about acid:
a) Proton donor
b) Turn litmus paper to blue
c) Sour taste
d) React with alkaline to give water and salt
e) None of the above
5. Pentose phosphate metabolism cycle is important to the cell because it give:
a) Acetyl CoA
b) ADP
c) ATP
d) NADH
e) NADPH
6. Hydrolysis of sucrose result:
a) Only glucose
b) Galactose
c) Fructose + glucose
d) Glucose + Glucose
e) Maltose + Glucose
7. Consider ketose:
a) Glucose
b) Fructose
c) Mannose
8. Presence of one of this substance in urine detect abnormal condition;
a) Calcium
b) Phosphates
c) Glucose
d) Urates
9. In cardiac infraction one of the measurements is not benefited: Creatinine
10. One of the following has highest conc. Of cholesterol:
a) Chylomicron
b) LDL
c) HDL
d) VLDL
e) Apolipoprtein
11. Hyper secretion of insulin cause: Hypoglycemia
12. Hyperglycemia hormone: Glucagon
13. Pyrimidin base:
a) Adenine
b) Guanine
c) Thymine
d) Uric acid
e) Urea
14. One use for measure abnormality in kidney: Creatinine
15. Does not stimulate with slight hemolysis: Cholesterol
16. One is stimulate on serum calcium level: Prolonged application of tourniquet on arm
17. Glycolysis done in :
a) Nucleus
b) Endoplasmic reticulum
c) Mitochondrion
d) Cytoplasm
e) None of the above
18. One of the above not present in Diabetic mellitus coma :
a) Hyperglycemia
b) Hypercholesterolemia
c) High number of ketene in urine
d) None of the above
19. In which substance give blood glucose when hydrolysis :
a) Muscle glycogen
b) Liver glycogen
c) Heart glycogen
d) Unsaturated fat
e) Triglyceride
20. Urea is final product of catabolism of:
a) Amino acid
b) Triglyceride
c) Cholesterol
d) Polysaccharide
21. Alkaline phosphates is the important enzyme to detect function in :
a) Liver
b) Bone
c) Liver and bone
d) None of the above
22. One of the following anticoagulant used for blood glucose :
a) EDTA
b) Heparin
c) Sodium oxalate
d) Florida Oxalate
e) Sodium citrate
23. AST important enzyme to detect abnormality in:
a) Liver disease
b) Heart disease
c) Liver and heart disease
d) None of the above
24. The diabetic patient is going to comma when blood glucose is?
a) 120 mg/dl
b) 160 mg/dl
c) Less than 50mg/dl
25. Amylase value is high in the following disease?
a) salivary glands
b) pancreas diseases
c) All of the previous
26. One jaundice patient has yellow skin, his billrubin is:
a) 2.5mg/dl
b) 1.2mg/dl
c) 5.0mg/dl
27. Acute diabetic patient has?
a) blood sugar more than 150mg/dl
b) blood sugar more than 180mg/dl
c) Glucose & Acetone in urine
28. Diabetic patient has one of the following symptoms?
a) Acidosis
b) Alkalosis
c) Dryness
29. Insulin is regulating blood sugar by?
a) Increase the influx of glucose into cells
b) activate glycogenesis
c) All the previous
30. To check the intestinal efficacy the following test is done?
a) Pepsin
b) Lipase
c) stool fats
31. Comma of diabetic patient shows?
a) glucose greater than 200mg/dl
b) glucose less than 200mg/dl
c) glucose greater than 500mg/dl
32. Bone matrix can also be call?
a) vascular tissue
b) fibrous tissue
c) Osteon
33. Serum LDH is elevated in all the following except?
a) skeletal disease
b) cardiac/ hepatic diseases
c) renal disease
34. Elevated Sodium & Chloride is seen in?
a) shock
b) diabetic acidosis
c) Severe dehydration
35. Which test is better to diagnose chronic bile duct?
a) total bilirubin
b) S-GOT
c) ALP
36. With age the renal threshold for glucose?
a) Increase
b) Decrease
c) does not change
37. Causes of high serum bilirubin are?
a) overload on liver
b) haemolysis
c) all of the previous
38. One of the following enzymes is affected by hemolysis?
a) SGOT
b) SGPT
c) LDH
39. HbA1C of diabetic patient is important for?
a) he has to come fasting
b) short term follow up
c) Long term follow up
40. For glucose tolerance test?
a) collect 5 blood samples only
b) collect 5 urine samples only
c) collect 5 blood samples + 5 urine samples
41. One of the following heart enzymes is measured after 4-8hr of chest pain?
a) GOT
b) LDH
c) CPK
42. Light effects one of the following?
a) Glucose
b) Urea
c) Billrubin
43. One of the following is specific diagnostic liver enzyme?
a) GOT
b) LDH
c) GPT
44. One of the following is important before anesthesia?
a) alkaline phosphates
b) acidic phosphates
c) Pseudocholine esterase
45. Acid phosphate is?
a) heart enzyme
b) liver enzyme
c) Prostatic enzyme
46. In Uric acid estimation?
a) its affected by carbohydrate meal
b) no need for fasting
c) The patient has to come fasting
47. All of the following are affected by meal except?
a) Glucose
b) Albumin
c) Creatininine
48. Na+ is the main?
a) intra cellular anion
b) intra cellular cation ----- (K)
c) extra cellular cation
49. One of the following electrolytes is affected by hemolisis?
a) Ca
b) Mg
c) k
50. The best kidney function test?
a) Urea
b) total protein
c) creatinine clearance
51. For GTT in adults the dose of glucose is?
a) 50gram
b) 100gram
c) 75gram
52. For GTT in children the dose of glucose is?
a) 30mg
b) 15gram
c) 30gram
53. Random blood glucose gives an idea?
a) to justify blood glucose
b) fasting patient
c) blood glucose in urgent cases
54. Exogenous triglyceride is carried on?
a) VLDL
b) HDL
c) Chylomicron
55. Endogenous triglyceride is carried on?
a) LDL
b) HDL
c) VLDL
56. Harmful cholesterol is carried on?
a) HDL
b) VLDL
c) LDL
57. Useful cholesterol is carried on?
a) Chylomicron
b) LDL
c) HDL
58. For lipid investigation patient has to fast?
a) 4-6hr
b) 6-8hr
c) 12-14hr
59. Test Used To Diagnose Obstructive Bile Duct
a) Bilirubin
b) Biliverdin
c) Urobilinogen
d) none of the above
60. Which of the following is protected from light :
a) Bilirubin
b) Cholesterol
c) total protein (TP.)
d) bun (blood urea nitrogen)
61. All of the following can assess the liver function except :
a) AST
b) ALT
c) ALP
d) Creatinine
62. Purine Associated With
a) ADENINE
b) GUANINE
c) URIC ACID
d) RNA

MICROBIOLOGY

1. Bacteria can cause pathogenesis to human by:


a) Capsullar
b) Secret enzymes
c) Endogenous toxins
d) Exogenous toxins
e) All of the above
2. Function of pili of the bacteria:
a) Attachment to the host tissue
b) Movement
c) Reproduction (multiplication)
d) Engulf of food
e) All of the above
3. Bacteria motile gram-rods:
a) Shigella
b) Bordetela pertussis
c) Pseudomonas aeruginoea
d) Yersenia pestis
4. Strict aerobe bacteria: Mycobacterium
5. One is always non motile gram –ve rods:
a) Haemophilus Influenza
b) Shigella
c) E.coli
d) Salmonella
e) Borditella Pertusis
6. One is motile gram –ve rods:
a) Haemophilus Influenza
b) Bacillus anthracis
c) Pseudomonas aerogenes
d) Vibrio cholera
e) Yersinia Pestis
7. One is gram +ve oval (cocci):
a) Bacillus anthracis
b) Meningococci
c) Pneumococci
8. A child diagnosis show scarlet fever the cause is:
a) Streptococcus Pyogenes
b) Staphylococci aureus
9. Xylose lysine Deoxycholate (XLD) is:
a) Selective media
b) Simple media
c) Differential media
d) Enriched media
e) Enrichment
10. One is not a Romanowsky stain:
a) Fields stain
b) Gram stain
c) Geimsa stain
d) Leishman stain
11. Best sterilization of Nutrient media done by:
a) Hot air oven
b) Autoclaving
12. Org. (bacteria) arranged in Chinese litters:
a) Corynebacteria Diphtheria
b) Bacillus anthraces
13. Confirmatory test of streptococcus pneumonia:
a) Catalase
b) Optochine disc
c) Coagulase
d) Bile insolubility
e) Bacitracin
14. Gram +ve cocci arranged in groups (clusters):
a) Streptococci
b) Staphylococci
15. One is always oxidase +ve
a) Haemophilus Influenza
b) Pseudomonas aurogenase
16. Gram-v bacteria color :
a) Dark purple
b) Pale to dark red
c) Orange
17. Bacteria need dark field to detect: T.pallidium (Syphilis)
18. Citrate test assess:
a) Mycobacterias
b) Staph
c) Strept
d) Colistridia sp
19. Nutrient agar: Basic media
20. Disease caused by Pyogen stereptococcus: Scarlet fever.
21. Org cause bloody in stool: Shigella sp.
22. Z.N stain use for diagnose: Mycobacterium
23. Media use for differentiate between L.F and N.L.F: Maconky media
24. Use to clean slide:
a) Ethyl alcohol
b) Methyl alcohol
25. The following is ingredient of culture medium:
a) Meat extract
b) Minral salts
c) Agar
d) Peptone
e) None of the above
f) All of the above
26. The most source of ATP in cell is :
a) Mitochondria
b) Cytoplasm
c) Nucleus
d) Cell wall
27. H Ag present in :
a) Pilli
b) Capsule
c) Flagella
d) Cell membrane
28. Selective and Differential medium for entero pathogen is:
a) Chocolate agar
b) Blood agar
c) DCA medium
d) Meat extract agar
e) None of the above
29. One of these is prokaryotic cell:
a) Fungi
b) Bacteria
c) Entameba histolytica
d) All of the above
e) None of the above
30. One of these org gram +ve cocci arranged in pairs :
a) Staphylococci
b) Streptococci
c) Meningococci
d) Entrococci
31. Substance used in catalyse reaction: H2O2
32. The best sample for the culture of children paralysis virus is?
a) anal swab
b) blood culture
c) stool culture
33. All of the following is true for salmonella except?
a) Motile
b) produces H2S
c) oxidase positive
34. The best media for urine culture is?
a) blood agar
b) chocolate agar
c) CLED agar
35. The sterilization of autoclave is?
a) 85c for 30min
b) 150c for 30min
c) 121c for 15min
36. Shigella soni colored in maconkey & EMB?
a) Colorless
b) Red
c) pink
37. All the following bacteria are interobacter except?
a) E.coli
b) Proteus
c) None of the previous
38. All the following parameters affecting gram staining except?
a) use H2SO4
b) add absolute alcohol after washing
c) Delaying the dryness of the slide
39. Blood sample is used to diagnose?
a) C.tetani
b) C.diphteria
c) none from them
40. Serious that causes food poisoning?
a) staph albus
b) salmonella typhi
c) Salmonella enteritidis
41. Which of the following causes UTI & INDOL positive?
a) Klebsiella
b) Staphylococci
c) E.coli
42. Which of the following culture media is suitable for semi quantitative bacterial count in urine
samples
a) Mc Conkey agar
b) Blood agar
c) XLD medium
d) Mannitol salt agar
e) CLED medium
43. One of the following is capsulated bacteria?
a) Streptococci
b) E.coli
c) klebsiella Pneumonia
44. The following organisms are lactose fomenters except
a) E. coli
b) Enterobacter cloacae
c) Shigella sonnei
d) Proteus spp.
45. All is true about Enterobacteriaceae except :
a) They are …… hemolytic and sorbitol
b) Ferments Mannitol
c) Grow in Methylene blue medium
d) Inhibited growth with 6.5 % NaCl and/or at temperature
46. All are true of campylobacter jejuni or Which of the following statements about campylobacter
is false:
a) Gram negative curved bacilli
b) Slow growth
c) Grow on XLD medium
d) Arranged in pairs
47. Beta hemolysis is enhanced when group B Streptococci is streaked at an angle in blood agar
plate with :
a) Streptococci
b) Staph aureus
c) Micrococcus
d) Streptococcus epiedermidis
e) Corynebacterim diphteriae
48. All statements are true about proteus mirabilis and Proteus vulgaris except:
a) Oxidase negative & liquefies gelatin
b) Exhibits swarming on BAP and Mc Conkey's agar
c) Urease positive
d) KCN and N2S positive
e) Positive to INDOLE test
49. Klebsiella pneumonia- Capsulated
50. Some organisms are said to be pathogenic if they are containing the following features :
a) Coagulase
b) Catalase
c) Sugar
d) Antibodies
51. Sterilization is best done by: 121c 15 PSI for 15-20 minutes
52. Organism Soluble In Bile :
a) Staph
b) Streptococci
c) Pneumococci (streptococcus pneumonia)
d) Haemophilous influenza
53. Thayer-martin media is the choice for the isolation of the following organism : -
a) Pseudomonas Aeroginosa
b) Haemophilus influenza
c) Nisseria gonorrhea
54. Confirmatory test for Strept. Pneumoniae :
a) Optochin sensitivity disc
b) Bacitracin disc sensitivity
c) Bile solubility
d) Catalase test
55. Specimen suitable for microfilaria
a) TISSUE
b) BLOOD
c) FLUID
56. Which of the following organisms is an anaerobic bacterium :
a) Clostiridium spp.
b) Haemophilus
c) E. Coli
d) Yersinia enterocolitica
57. Nutrient agar is :
a) Selective media
b) Differential media
c) Special media
d) Basic medium
58. The Following Organisms Are Encapsulated Except :
a) Pseudomonas aeroginosa
b) E. Coli
c) Haemophilous influenza
d) Streptococcus pneumonia
59. Vi Ag is seen in :
a) CAPSULE
b) PILI
c) SPORE
d) FLAGELLA
60. All are true about enterococci except :- (Strept. Faecalis)
a) have carbohydrate antigen of group d streptococci
b) positive aesculin hydrolysis
c) grow in the presence of bile salts
d) Do not grow in the presence of 6.5 % Nacl nor at 45 c
61. Significant bacteruria , puria , and alkaline urine probably indicate urinary tract infection due to:
a) Staphylococcus epidermidis
b) E. Coli
c) Proteus vulgaris
d) Pseudomonas aerginosa
e) Enterococcus Faecalis
62. Streptococci responsible for the majority of human infections are :
a) Group A
b) beta-haemolytic streptococci
c) bacitracin-s and camp ( - )
d) all of these
e) none of these
63. To differentiate between nisseria gonorrhoea and nisseria meningitidis :
a) fermentation of dextrose and lactose
b) Fermentation of dextrose and maltose
c) fermentation of maltose and lactose
d) Fermentation of maltose and sucrose
64. Blood culture is indicated in the following bacterial diseases except
a) Meningitis
b) Endocarditis
c) Gastroenteritis
d) Pyelonephritis
e) Pneumonia
65. Which of the following statements about campylobacter jejuni is false
a) gram negative curved bacilli
b) Arranged in pairs (see-gull)
c) slow growth
d) Grows on XLD medium
incubation temperature at 42 c
66. The invasiveness of streptococcus pneumoniae is due to the production of:
a) Haemolsins
b) Endotoxins
c) Extotoxins
d) Polysaccharide capsule
67. Agar is characterized by all the following except?
a) freezing point is 42c & melting point is 100c
b) nutritive
c) un-nutritive

PARASITOLOGY

1. Infective stage of Entrobius Vermicularis is:


a) Larva
b) Cercaria
c) Egg
d) Cyst
e) Metacercaria
2. The definitive host is an animal:
a) Entamoeba histolitica
b) Toxoplasma Gondii
c) Trypanosoma cruzi
d) E. coli
e) Giardia lamblia
3. Parasite can cause fetal cerebral infection:
a) Taenia
b) Toxoplasma gondii
4. Intracellular parasite:
a) E.coli
b) Toxoplasma gondii
c) Giaradia lamblia
d) Typanosoma cruzi
e) Trichomonas vaginalis
5. Non motile protozoa:
a) E.coli
b) Entamoeba Histolytica
c) Trypanosoma cruzi
d) Giardia lamblia
e) Trichomonas vaginalis
6. Identification of Leishmania parasite: Lymph node aspirate.
7. Infective stage of Taenia:
a) Larva
b) Cercaria
c) Embrocated egg
d) Cyst
e) Metacercaria
8. Autoinfection of human can caused in case of:
a) Entrobius Vermicularis
b) Taenia Sp.
c) Anclystoma Dudenal
d) Ascaris Lumbercoidis
9. Iodine stain of Entamoeba cyst used to detect (diagnose):
a) Chromatoidal bars
b) Cell wall
c) Nuclei
10. Diagnostic stage of Ascaris lumbericoidis: Finding of a typical fertilized egg in stool
11. Parasite can be detected in vaginal and urethral discharge: Trichomonas Vaginalis
12. Blood of Microfilaria infection diagnostic with: Wuchereria bancrofti
13. Oocyst benefited to diagnose:
a) E.histolytica
b) E.coli
c) Toxoplasma gondii
d) Giaradia lamblia
e) Typanosoma cruzi
14. Cutaneous Leishmanias is transmitted by: Phlebotomus sand fly
15. Shistosoma hematobium diagnosis: Egg in urine with lateral spine
16. One of these organism multiply intracellular parasite:
a) Gardia lamblia
b) Entameba histolytica
c) E. coli
d) Tryponosoma Cruzi
e) Toxoplasma gondii
17. infective stage of Ascaris lumbricodes :
a) Larva
b) Cercaria
c) Egg
d) Cyst
e) Metacercaria
18. The worm that causes blood in urine?
a) Ascaris
b) Fasciola
c) Schistosoma haematobium
19. Generally diagnosed by recovery & identification of typical larva in stool?
a) hook worms
b) t.trichura
c) s.stercoralis
20. Polio myeletis is transmitted through?
a) Skin
b) Respiration
c) Feco-oral
21. Which of the following is not laying eggs in small intestine?
a) hook worm
b) t.saginata
c) Pin worm
22. A parasite which does not deposit eggs in the intestine
a) E. vermicularis (Pin Worm)
b) A. duodenale
c) A. lumbricoides
d) T.saginata
e) T. spiralis
23. Best for identification of parasite if you cannot see it in stool ( Negative in stool ): Concentration
Technique
24. The causative parasite of Egyptian Bilharisiasis
a) Trichomonas Vaginalis
b) Schitosoma haematobium
c) Schitosoma japonicum
d) Schitosoma mansoni
e) All of the above
25. Identification of fertilized eggs in feces: Ascaris Lumbricoides
26. Sugar cannot be detected in urine using copper reduction test
a) Fructose
b) Galactose
c) Arabinose
d) Sucrose
27. The infective stage of Ancylostoma Duodenale is :
a) Eggs in stool
b) larva in stool
c) larva in tissue
d) cysticercous bovis
28. What practical Cardiolipin is applicable in laboratory work?
a) FTA
b) VDRL & RPR
c) RPR ONLY
d) VDRL ONLY
29. The intermediate host of Toxoplasma is :
a) Dog
b) Cat
c) Mouse
d) All
e) None
30. A semen specimen, contain sugar source of the spermatozoa energy is:
a) Glucose
b) Mannose
c) Glactose
d) Fructose
e) Sucrose
27. T0 check the intestinal efficacy the following test is done?
a) Pepsin
b) Lipase
c) Fats stool

IMMUNOLOGY

1. The smallest molecular weight Ig is:


a) IgG
b) IgM
c) IgD
d) IgE
e) IgA
2. B-lymphocyte after attack foreign bodies ,called Plasma cell
3. One of the immunoglobulin present in trace amount in serum :
a) IgG
b) IgE
c) IgM
d) IgD
e) IgA
4. Immunoglobulin is?
a) Beta globulin
b) Alpha globulin
c) Gamma globulin
5. Ig ( Immunoglobulin ) Increased In Case Of Parasitic Infection & Allergy
a) IgG
b) IgA
c) IgM
d) IgE
6. (HEAVIER ) 1St?
a) IgM
b) IgA
c) IgE
7. Ig can cause HDN :
a) IgM
b) IgA
c) IgE
d) IgG
8. Ig not part of acquired immunity :
a) IgM
b) IgA
c) IgE
d) IgG
9. One of the immunoglobulin present in trace amount in serum :
a) IgG
b) IgE
c) IgM
d) IgD
e) IgA

ELISA

1. A standard micro plates in ELISA test has?

a) 98 wells
b) 94 wells
c) 96 wells

2. The enzyme in ELISA testing is present in the?

a) Buffer
b) micro plate
c) conjugate

3. Antigen antibody complex are?

a) weakly bound
b) no bounds
c) strongly bound

4. Washing must be done in all heterogeneous ELISA technique because?

a) increase the specificity


b) increase the sensitivity
c) it removes the excess binding

5. The label in ELISA tests is?

a) radio active substance


b) antibody
c) enzyme

6. method used to detect hormone in the laboratory : ELIZA

BLOOD BANK

1. Blood medium commonly used in blood bank:


a) Citrate Phosphate Dextrose adenine (CPD-A)
b) EDTA anticoagulant
c) Heparin anticoagulant
d) Saline alanin glucose maltose (SAGM)
2. Indirect anti-antibody test used to detect: (Indirect Combs Test)
a) Sensitized RBCs in patient blood
b) IgG
c) IgM
d) Sensitized antibody in patient serum
e) None of the above
3. Sensitized Ab in serum detect by: Indirect antiglublin test
4. Direct anti-antibody test used to detect: (Direct Combs Test)
a) Sensitized RBCs in patient blood
b) IgG
c) IgM
d) Sensitized antibody in patient serum
e) None of the above
5. Anti-Human Immunoglubin is:
a) Coombs reagent
b) Anti-Ab
c) IgG
d) All of the above
e) None of the above
6. Rh-ve patient means he does not have:
a) E antigen
b) D antigen
c) B antigen
d) C antigen
e) None of the above
7. Life span of red blood cells in the circulation (body):
a) 80 days
b) 120 day
c) 130 day
d) 20 day
e) 7 days
8. Use O cell tube on lab for:
a) As control
b) To detect Ag
c) To detect Ab
9. All cause false –v ABO blood grouping except: Too short incubation
10. Store FFPs at -80oC:
a) 1year
b) 1month
c) 4 year
d) 2 year
e) 6year
11. In Rh-v person , this Ag not found: D- Antigen
12. In O+v person
a) No presence of Ag
b) No presence of Ab
13. CPD_A anticoagulant used to store blood for:
a) 35 days
b) 45 days
c) 5 days
d) 15 days
e) None of the above
14. In Anti human globulin test do wash of RBC because all serum contain :
a) Albumin
b) a_glubulin
c) b_ globulin
d) fibrinogen
e) Immunoglobulin
15. Reason of false positive ABO in compatibility: Cold agglutination
16. In O blood group the Ab in serum is :
a) Anti A1B
b) Anti A
c) Anti B
d) None of the above
e) All of the above
17. Blood donor selection in KSA are all of the following except?
a) Hb% 12.5-16
b) free from syphilis
c) One year after delivery
18. Blood transfusion can transmit?
a) HIV
b) CMV
c) All of the previous
19. Causes false negative ABO incomparability/ cross matching :
a) Deteriorated reagent
b) Not putting antisera
c) Under incubation
20. False positive ABO incompatibility/cross matching: Cold Agglutinin
21. Why do we have to wash red blood cells in cross matching: To remove an excess antibody
present in the sample
22. Antibody found in blood group A individuals :
a) ANTI-A
b) ANTI-AB
c) ANTI-B
23. To differentiate anti i from anti I
a) A1 CELLS
b) A2 CELLS
c) Cord Cells
d) None of the above
24. Cross-Match in (Major X-Matching): Donor Cells + Patient Serum

HORMONE

1. Function of Thyroxin is:


a) Increase Oogenesis
b) Stimulate contraction of uterus
c) Increase basal metabolic
2. Increase in Thyroxin Stimulating Hormone (TSH) causes:
a) Cushing disease
b) Gigantism
c) Exophthalamic goiter
d) Hypoglycemia
3. Function of Estrogen:
a) Increase Oogenesis
b) Stimulate contraction of uterus
c) Increase basal metabolic
4. Increase in growth hormone causes:
a) Cushing disease
b) Gigantism
c) Exophthalamic goiter
d) Hypoglycemia
5. Immunoglobulin pregnancy test related to:
a) IgG
b) IgM
c) Human chorionic Gonadotropin
6. Target of Prolactin in female:
a) Ovary
b) Mammary glands
7. Hyper secretion of cortisol cuase: Cushing disease
8. Hyperparathyroidism hormone cause:
a) Elevated ca level
b) Decrease ca level
9. Regulation of calcium level in serum by:
a) Calcitonine hormone only
b) Parathyroid hormone only
c) Calcionine with parathyroid hormone
d) Vitamin D
e) PTH+Vitamin D+calcitonin
10. Oxytocine hormone function :
a) Increase Oogenesis
b) Stimulate contraction of uterus
c) Increase basal metabolic
11. Function of Cortisol :
a) Increase Oogenesis
b) Stimulate contraction of uterus
c) Increase basal metabolic
12. While using the pregnancy test we are measuring?
a) total HCG
b) beta HCG &LH
c) beta HCG
13. ADH is secreted from?
a) thyroid gland
b) anterior pituitary
c) Posterior pituitary
VIROLOGY

1. An HIV-positive patient asks you if you can tell him the chances of him progressing to symptomatic
AIDS. Which one of the following tests would be most useful?

a. CD4 lymphocyte count


b. HIV antibody test
c. HIV RT PCR
d. Neopterin
e. HIV p24 antigen

HIV RT PCR, a nucleic acid amplification test for HIV RNA, has recently been shown to be the most
valuable test for a) monitoring a patient’s progress during triple drug therapy and b) determining the
chances of progression to AIDS. A viral load of 750,000 copies per ml significantly increases the chance
of progression to AIDS within 5 years. The other tests listed do not accurately predict progression to
AIDS.

2. Which of the following viruses causes an acute febrile rash and produces disease in
immunocompetent children but has been associated with transient aplastic crises in persons with sickle
cell disease?

a. Rubeola
b. Varicella-zoster
c. Parvovirus
d. Rubella
e. Herpes simplex

Parvovirus B 19 is the causative agent of erythema infectiosum (fifth disease). It is associated with
transient aplastic crisis in persons with hereditary hemolytic anemia. In adults, it is also associated with
polyarthralgia.

3. Infection with herpes simplex virus, a common human pathogen, is best described by which of the
following statements?

a. The CNS and visceral organs are usually involved

b. It rarely recurs in a host who has a high antibody titer

c. It can be reactivated by emotional disturbances or prolonged exposure to sunlight

d. Initial infection usually occurs by intestinal absorption of the virus

e. Infection with type 1 virus is most common


4. The latest and most effective therapy for AIDS patients includes azidothymidine (AZT), dideoxyinosine
(DDI), and saquinavir or similar agents. Use of these three drugs would inhibit which of the following
viral processes?

a. RNase, DNase

b. gp120 formation

c. p24 antibody expression

d. All membrane synthesis

e. Reverse transcriptase, protease

5. An HIV-positive patient prior to being treated with AZT, DDI, and saquinavir has a CD4 lymphocyte
count and an HIV RNA viral load test done. Results are as follows:

CD4: 50 CD4 lymphocytes per microliter

HIV RNA: 750,000 copies per ml

Which of the following statements best describes the above patient?

a. This patient is no longer in danger of opportunistic infection

b. The 5-year prognosis is excellent

c. The patientâ s HIV screening test is most likely negative

d. The patient is not infectious

e. The viral load of 750,000 copies per ml suggests that the patient will respond to triple therapy

6. This HIV-positive patient with a viral load of 750,000 copies of HIV RNA/ml and a total CD4 count of 50
is at an increased risk for a number of infectious diseases. For which of the following diseases is the
patient at no more added risk than an immunocompetent host?

a. Pneumocystic pneumonia

b. Mycobacterial disease

c. Kaposiâ s sarcoma

d. Pneumococcal pneumonia

e. Herpes simplex virus

7. Infectious mononucleosis, a viral disorder that can be debilitating, is characterized by which of the
following statements?
a. It is most prevalent in children less than 14 years old

b. It is caused by a rhabdovirus

c. The causative pathogen is an Epstein-Barr virus

d. Affected persons respond to treatment with the production of heterophil antibodies

e. Ribavirin is the treatment of choice

8. A tube of monkey kidney cells is inoculated with nasopharyngeal secretions. During the next 7 days,
no cytopathic effects (CPEs) are observed. On the eighth day, the tissue culture is infected accidentally
with a picornavirus; nevertheless, the culture does not develop CPEs. The most likely explanation of this
phenomenon is that

a. The nasopharyngeal secretions contained hemagglutinins

b. The nasopharyngeal secretions contained rubella virus

c. Picornavirus does not produce CPEs

d. Picornavirus does not replicate in monkey kidney cells

e. Monkey kidney cells are resistant to CPEs

9. The clinical picture of arbovirus infection fits one of three categories: encephalitis, hemorrhagic fever,
or fever with myalgia. One of the characteristics of arboviruses is that they

a. Are transmitted by arthropod vectors

b. Are usually resistant to ether

c. Usually cause symptomatic infection in humans

d. Are closely related to parvoviruses

10. Which one of the following statements best describes interferonâ s suspected mode of action in
producing resistance to viral infection?

a. It stimulates a cell-mediated immunity

b. It stimulates humoral immunity

c. Its direct antiviral action is related to the suppression of messenger RNA formation

d. Its action is related to the synthesis of a protein that inhibits translation or transcription

e. It alters the permeability of the cell membrane so that viruses cannot enter the cell
11. Coronaviruses are recognized by club-shaped surface projections that

are 20 nm long and resemble solar coronas. These viruses are characterized

by their ability to

a. Infect infants more frequently than adults

b. Cause the common cold

c. Grow well in the usual cultured cell lines

d. Grow profusely at 50ï °C

e. Agglutinate human red blood cells

12. Delta hepatitis only occurs in patients who also have either acute or

chronic infection with hepatitis B virus. The delta agent is

a. An incomplete hepatitis B virus

b. Related to hepatitis A virus

c. A hepatitis B mutant

d. An incomplete RNA virus

e. Hepatitis C

13. Which of the following antiviral agents is a purine nucleoside analogue

that has shown promise with Lassa fever, influenza A and B, and respiratory

syncytial virus (RSV)?

a. Amantadine

b. Rimantadine

c. Vidarabine

d. Ribavirin

e. Acyclovir

14. Echoviruses are cytopathogenic human viruses that mainly infect the

a. Respiratory system

b. Central nervous system

c. Blood and lymphatic systems

d. Intestinal tract
e. Bladder and urinary tract

15. The most sensitive test for the diagnosis of herpes simplex (HSV)

meningitis in a newborn infant is

a. HSV IgG antibody

b. HSV polymerase chain reaction (PCR)

c. HSV culture

d. Tzanck smear

e. Cerebrospinal fluid (CSF) protein analysis

8 Microbiology

16. Acute hemorrhagic conjunctivitis (AHC) is a contagious ocular infection

characterized by pain, swelling of the eyelids, and subconjunctival

hemorrhages. AHC has been reported to be caused by which of the following

viruses?

a. Coronavirus

b. Reovirus

c. Rhinovirus

d. Enterovirus

e. Respiratory syncytial virus

17. Mumps virus accounts for 10 to 15% of all cases of aseptic meningitis

in the United States. Infection with mumps virus

a. Is apt to recur periodically in many affected persons

b. Will usually cause mumps orchitis in postpubertal males

c. Is maintained in a large canine reservoir

d. Usually produces severe systemic manifestations

e. Is preventable by immunization

18. The serum of a newborn infant reveals a 1:32 cytomegalovirus (CMV)

titer. The child is clinically asymptomatic. Which of the following courses

of action would be advisable?

a. Repeat the CMV titer immediately


b. Wait 6 months and obtain another titer on the baby

c. Obtain a CMV titer from all siblings

d. Obtain an anti-CMV IgM titer from the mother

e. Obtain an anti-CMV IgM titer from the baby

19. A 3-year-old child presents at the physicianâ s office with symptoms of

coryza, conjunctivitis, low-grade fever, and Koplikâ s spots. The causative

agent of this disease belongs to which group of viruses?

a. Adenovirus

b. Herpesvirus

c. Picornavirus

d. Orthomyxovirus

e. Paramyxovirus

Virology 9

20. One of the most common sexually transmitted diseases that may lead

to cervical carcinoma is caused by which of the following viruses?

a. Cytomegalovirus

b. Papillomavirus

c. Epstein-Barr virus

d. Herpes simplex virus

e. Adenovirus

21. Which virus is the leading cause of the croup syndrome in young children

and, when infecting mammalian cells in culture, will hemabsorb red

blood cells?

a. Group B coxsackievirus

b. Rotavirus

c. Parainfluenza virus

d. Adenovirus

e. Rhinovirus

22. Hepatitis E, a recently characterized hepatitis virus, is best described


by which of the following statements?

a. It is not a threat to the blood supply

b. It is a major cause of blood-borne hepatitis

c. It is prevalent in North America

d. It is a single-stranded DNA virus

e. The disease resembles hepatitis C

23. Meningitis is characterized by the acute onset of fever and stiff neck.

Aseptic meningitis may be caused by a variety of microbial agents. During

the initial 24 h of the course of aseptic meningitis, an affected personâ s cerebrospinal

fluid is characterized by

a. Decreased protein content

b. Elevated glucose concentration

c. Lymphocytosis

d. Polymorphonuclear leukocytosis

e. Eosinophilia

10 Microbiology

24. Infection with hepatitis D virus (HDV; delta agent) can occur simultaneously

with infection with hepatitis B virus (HBV) or in a carrier of hepatitis

B virus because HDV is a defective virus that requires HBV for its

replicative function. What serologic test can be used to determine whether

a patient with HDV is an HBV carrier?

a. HBsAg

b. HBc IgM

c. HBeAg

d. HBs IgM

e. HBs IgG

25. A nurse develops clinical symptoms consistent with hepatitis. She

recalls sticking herself with a needle approximately 4 months before after

drawing blood from a patient. Serologic tests for HBsAg, antibodies to


HBsAg, and hepatitis A virus (HAV) are all negative; however, she is positive

for IgM core antibody. The nurse

a. Does not have hepatitis B

b. Has hepatitis A

c. Is in the late stages of hepatitis B infection

d. Is in the â windowâ (after the disappearance of HBsAg and before the appearance

of anti-HBsAg)

e. Has hepatitis C

26. Eastern equine encephalitis virus is associated with a high fatality rate.

Control of the disease could be possible by eradication of

a. Horses

b. Birds

c. Mosquitoes

d. Fleas

e. Ticks

27. Adults who have had varicella as children occasionally suffer a recurrent

form of the disease, shingles. The agent causing these diseases is a

member of which of the following viral families?

a. Herpesvirus

b. Poxvirus

c. Adenovirus

d. Myxovirus

e. Paramyxovirus

Virology 11

28. Rhinovirus is primarily transmitted by

a. Droplet aerosolization

b. Sexual activity

c. Fecal-oral route

d. Fomites
e. Vertical transmission from mother to child

29. German measles virus (rubella), a common cause of exanthems in

children, is best described by which of the following statements?

a. Measles (rubeola) and German measles (rubella) are caused by the same virus

b. Incubation time is approximately 3 to 4 weeks

c. Vesicular rashes are characteristic

d. Onset is abrupt with cough, coryza, and fever

e. Specific antibody in the serum does not prevent disease

30. The presence of Negri inclusion bodies in host cells is characteristic of

a. Mumps

b. Infectious mononucleosis

c. Congenital rubella

d. Aseptic meningitis

e. Rabies

31. Kuru is a fatal disease of certain New Guinea natives and is characterized

by tremors and ataxia; Creutzfeldt-Jakob disease (CJD) is characterized

by both ataxia and dementia. These diseases are thought to be caused

by

a. Slow viruses

b. Cell wallâ deficient bacteria

c. Environmental toxins

d. Prions

e. Flagellates

32. According to recommendations issued by the U.S. Public Health Service,

which of the following statements regarding vaccination against

smallpox is true?

a. Pregnant women should be vaccinated in the first trimester

b. Persons who have eczema should be vaccinated soon after diagnosis

c. Persons who have immune deficiencies should be vaccinated every 5 years


d. Persons traveling abroad need not be vaccinated

e. Children should be vaccinated before they begin school

12 Microbiology

33. Hepatitis D virus (delta agent) is a defective virus that can replicate

only in cells already infected with which of the following viruses?

a. Hepatitis A virus

b. Epstein-Barr virus

c. Hepatitis G virus

d. Hepatitis B virus

e. HIV

34. A patient presents with keratoconjunctivitis. The differential diagnosis

should include infection with which of the following viruses?

a. Parvovirus

b. Adenovirus

c. Epstein-Barr virus

d. Respiratory syncytial virus

e. Varicella-zoster virus

35. A hospital worker is found to have hepatitis B surface antigen. Subsequent

tests reveal the presence of e antigen as well. The worker most likely

a. Is infective and has active hepatitis

b. Is infective but does not have active hepatitis

c. Is not infective

d. Is evincing a biologic false-positive test for hepatitis

e. Has both hepatitis B and C

36. Alphavirus causes which one of the following viral diseases?

a. Marburg virus disease

b. St. Louis encephalitis

c. Western equine encephalitis

d. Dengue
e. Yellow fever

37. Several antiviral compounds have been developed during the last

decade. One such compound is ribavirin, a synthetic nucleoside structurally

related to guanosine. Ribavirin therapy has been successfully used

against

a. Respiratory syncytial virus

b. Herpes simplex virus

c. Hepatitis B

d. Group A coxsackievirus

e. Parvovirus

Virology 13

38. An immunocompromised person with history of seizures had an MRI

that revealed a temporal lobe lesion. Brain biopsy results showed multinucleated

giant cells with intranuclear inclusions. The most probable cause of

the lesion is

a. Hepatitis C virus

b. Herpes simplex virus

c. Listeria monocytogenes

d. Coxsackievirus

e. Parvovirus

39. Which of the following procedures or clinical signs is most specific for

the diagnosis of infectious mononucleosis caused by the Epstein-Barr

virus?

a. Laboratory diagnosis is based on the presence of â atypical lymphocytesâ and

EBV-specific antibody

b. Growth in tissue culture cells

c. Heterophile antibodies in serum

d. Lymphadenopathy and splenomegaly on physical examination

e. B-cell lymphocyte proliferation


40. An infant, seen in the ER, presents with a fever and persistent cough.

Physical examination and a chest x-ray suggest pneumonia. Which of the

following is most likely the cause of this infection?

a. Rotavirus

b. Adenovirus

c. Coxsackievirus

d. Respiratory syncytial virus

e. Rhinovirus

41. Which one of the following groups of people may be at increased risk

for HIV infection?

a. Members of a household in which there is a person who is HIV-positive

b. Receptionists at a hospital

c. Factory workers whose coworkers are HIV-positive

d. Foreign service employees who are hospitalized in Zaire for bleeding ulcers

e. Homosexual females

14 Microbiology

42. An obstetrician sees a pregnant patient who was exposed to rubella

virus in the eighteenth week of pregnancy. She does not remember getting

a rubella vaccination. The best immediate course of action is to

a. Terminate the pregnancy

b. Order a rubella antibody titer to determine immune status

c. Reassure the patient because rubella is not a problem until after the thirtieth

week

d. Administer rubella immune globulin

e. Administer rubella vaccine

43. Mad Cow Disease has been highly publicized in Great Britain. This

disease, which is similar to scrapie, is caused by

a. A prion

b. A virus
c. Rickettsiae

d. An autoimmune reaction

e. A bacterium with a defective cell wall

44. A patient has all the gastrointestinal symptoms of infection with

hepatitis A virus (HAV), yet all the tests for HAV-IgG and HAV-IgM are nonreactive.

A possible cause of this infection is

a. Hepatitis B surface antigen

b. Hepatitis C

c. Hepatitis D

d. Hepatitis E

e. Rotavirus

45. A 70-year-old nursing home patient refused the influenza vaccine and

subsequently developed influenza. She died of acute pneumonia 1 week

after contracting the â flu.â The most common cause of acute postinfluenzal

pneumonia is

a. Legionella

b. Listeria

c. Staphylococcus aureus

d. Klebsiella

e. Escherichia coli

Virology 15

46. Which of the following viruses is primarily transmitted by the fecaloral

route?

a. St. Louis encephalitis virus

b. Colorado tick fever virus

c. Coxsackievirus

d. Yellow fever virus

e. Dengue fever virus

47. Hantavirus is an emerging pathogen that is best described by which of


the following statements?

a. Influenza-like symptoms are followed rapidly by acute respiratory failure

b. Hemolysis is common in infected patients

c. It is acquired by inhalation of aerosols of the urine and feces of deer

d. Transmission from human to human is common

e. There is effective antiviral therapy available

48. Erythema infectiosum (fifth disease), a self-limited disease of children,

is caused by

a. Measles

b. Parvovirus

c. Rubella

d. Human herpesvirus type 6

e. Norwalk virus

49. Which one of the following viruses may be human tumor virus?

a. Epstein-Barr virus (EBV)

b. HIV

c. Papillomavirus

d. Varicella-zoster virus (VZV)

e. Herpes simplex virus, type 2 (HSV)

50. Parvovirus infection, the cause of a mild exanthem in children, is

characterized by

a. Epidemic acute respiratory disease

b. Gastroenteritis

c. Whooping coughâ like disease

d. Keratoconjunctivitis

e. Acute hemolytic anemia

16 Microbiology

51. Cytomegalovirus (CMV) infection is common. Which one of the following

statements best characterizes CMV?


a. It can be transmitted across the placental barrier

b. While a common infection, CMV is almost always symptomatic

c. The CMV can be cultured from red blood cells of infected patients

d. Unlike other viral infections, CMV is not activated by immunosuppressive therapy

e. There is no specific therapy for CMV

52. Human rotaviruses are characterized by which of the following statements?

a. They produce an infection that is primarily seen in adults

b. They produce cytopathic effects in many conventional tissue culture systems

c. They are lipid-containing RNA viruses possessing a double-shelled capsid

d. They can be sensitively and rapidly detected in stools by the enzyme-linked

immunosorbent assay (ELISA) technique

e. They have been implicated as a major etiologic agent of infantile respiratory disease

53. Subacute sclerosing panencephalitis virus (SSPE) is best described by

which of the following statements?

a. It is a progressive disease involving both white and gray matter

b. It is a late CNS manifestation of mumps

c. It is a common event occurring in 1 of 300,000 cases of mumps

d. Viral DNA can be demonstrated in brain cells

e. Demyelination is characteristic

54. Rotavirus is a double-stranded RNA virus with a double-walled capsid.

Which one of the following statements best describes rotavirus?

a. There are no related animal viruses

b. It is a major cause of neonatal diarrhea

c. It is readily cultured from the stool of infected persons

d. Maternal antibody does not appear to be protective

e. Early breast-feeding offers no protection to neonates against it

Virology 17

55. Paramyxoviruses are most commonly associated with which of the following

diseases?
a. Fifth disease

b. Rubella

c. Croup

d. Tonsillitis

e. Otitis media

56. Human papillomavirus is most commonly associated with

a. Rectal polyps

b. Prostate cancer

c. Condyloma acuminatum

d. Hepatic carcinoma

e. Carcinoma of the lung

57. Reverse transcriptase is an enzyme unique to the retroviruses. Which

one of the following is a function of the enzyme reverse transcriptase?

a. DNase activity

b. RNA-dependent RNA polymerase activity

c. RNA isomerase activity

d. RNA-dependent DNA polymerase activity

e. Integration activity

58. St. Louis encephalitis, a viral infection, was first recognized as an

entity in 1933. Which of the following best describes SLE?

a. It is transmitted to humans by the bite of an infected tick

b. It is caused by a togavirus

c. It is the major arboviral cause of central nervous system infection in the United

States

d. It may present initially with symptoms similar to influenza

e. Laboratory diagnosis is routinely made by cultural methods

18 Microbiology

59. There is considerable overlap of signs and symptoms seen in congenital

and perinatal infections. In a neonate with â classicâ symptoms of congenital


cytomegalovirus (CMV) infection, which one of the following tests

would be most useful in establishing a diagnosis?

a. CMV IgG titer on neonateâ s serum at birth

b. CMV IgG titer on motherâ s serum at birth of infant

c. CMV IgM titer on neonateâ s serum at birth and at 1 month of age

d. Total IgM on neonateâ s serum at birth

e. Culture of motherâ s urine

60. Interferon, a protein that inhibits viral replication, is produced by cells

in tissue culture when the cells are stimulated with which of the following?

a. Botulinum toxin

b. Synthetic polypeptides

c. Viruses

d. Chlamydiae

e. Gram-positive bacteria

61. Which one of the following statements best describes the cytopathic

effects of viruses on host cells?

a. Usually morphological in nature

b. Often associated with changes in mitochondrial membranes

c. Pathognomonic for an infecting virus

d. Rarely fatal to the host cell

e. Can only be seen with an electron microscope

62. A 17-year-old girl presents with cervical lymphadenopathy, fever, and

pharyngitis. Infectious mononucleosis is suspected. The most rapid and

clinically useful test to make this diagnosis is

a. IgM antibody to viral core antigen (VCA)

b. IgG antibody to VCA

c. Antibody to Epstein-Barr nuclear antigen (EBNA)

d. Culture

e. C reactive protein (CRP)


Virology 19

63. Which one of the following viruses would be most likely to establish a

latent infection?

a. Adenovirus

b. Measles virus

c. Influenza virus

d. Parvovirus

e. Coxsackievirus group B

64. A regimen that includes appropriately administered gamma globulin

may be contraindicated in which one of the following diseases?

a. Hepatitis A

b. Hepatitis B

c. Rabies

d. Poliomyelitis

e. Infectious mononucleosis

65. Atypical lymphocytosis is most likely to be found in which one of the

following diseases?

a. Encephalitis caused by herpes simplex virus (HSV)

b. Mononucleosis induced by Epstein-Barr virus

c. Parvovirus infection

d. Chronic hepatitis C

e. Rotavirus gastroenteritis

66. A patient has arthralgia, a rash, lymphadenopathy, pneumonia but no

fever. Which of the following diseases is most likely based on these symptoms?

a. Dengue fever

b. St. Louis encephalitis

c. Infectious mononucleosis

d. Hepatitis

e. HIV infection
20 Microbiology

67. Hepatitis C (HCV) is usually a clinically mild disease, with only minimal

elevation of liver enzymes. Hospitalization is unusual. Which one of

the following statements best characterizes HCV?

a. Few cases progress to chronic liver disease

b. It often occurs in posttransfusion patients

c. HBV but not HCV infections occur in IV drug abusers

d. It is a DNA virus

e. Blood products are not tested for antibody to HCV

68. Which of the following markers is usually the first viral marker

detected after hepatitis B infection?

a. HBeAg

b. HBsAg

c. HBcAg

d. Anti-HBc

e. HbeAb

69. Which of the following may be the only detectable serological marker

during the early convalescent phase of HBV infection (window phase)?

a. HBeAg

b. HBsAg

c. HBcAg

d. Anti-HBc

e. HbeAb

70. Which one of the following markers is closely associated with HBV

infectivity and DNA polymerase activity?

a. HBeAg

b. HBsAg

c. HBcAg

d. Anti-HBc
e. HBeAb

71. Which of the following is found within the nuclei of infected hepatocytes

and not usually in the peripheral circulation?

a. HBeAg

b. HBsAg

c. HBcAg

d. Anti-HBc

e. HbeAb

Virology 21

72. Which one of the following viruses is the leading cause of congenital

malformations?

a. Rabies

b. Rhinovirus

c. Cytomegalovirus

d. Respiratory syncytial virus

e. Mumps

73. Orchitis, which may cause sterility, is a possible manifestation of

which of the following?

a. Rabies

b. Rhinovirus

c. Cytomegalovirus

d. Respiratory syncytial virus

e. Mumps

74. Which of the following is a leading cause of pneumonia primarily in

infants?

a. Rabies

b. Rhinovirus

c. Cytomegalovirus

d. Respiratory syncytial virus


e. Mumps

75. Which of the following causes a fatal encephalitis for which a vaccine

is available?

a. Rabies

b. Rhinovirus

c. Cytomegalovirus

d. Respiratory syncytial virus

e. Mumps

76. Traditional vaccination for the common cold is virtually impossible

because there are multiple serotypes of which one of the following viruses?

a. Rabies

b. Rhinovirus

c. Cytomegalovirus

d. Respiratory syncytial virus

e. Mumps

22 Microbiology

77. Which of the following is available and effective for hepatitis A?

a. Acyclovir

b. Killed virus vaccine

c. Inactivated virus vaccine

d. Live virus vaccine

e. Recombinant viral vaccine

78. Patients should be vaccinated annually for influenza with which of the

following vaccines?

a. Immune serum globulin

b. Killed virus vaccine

c. Inactivated virus vaccine

d. Live virus vaccine

e. Recombinant viral vaccine


79. The vaccine for measles is best characterized as a

a. Bacterin

b. Killed virus vaccine

c. Inactivated virus vaccine

d. Live virus vaccine

e. Recombinant viral vaccine

80. Which one of the following would be the treatment of choice for HSV

infection?

a. Acyclovir

b. Killed virus vaccine

c. Herpes immune globulin

d. Azythromycin

e. Recombinant viral vaccine

81. Which of the following best describes the presently available vaccine

for hepatitis B?

a. Synthetic peptide vaccine

b. Killed virus vaccine

c. Inactivated virus vaccine

d. Live virus vaccine

e. Recombinant viral vaccine

Virology 23

82. Chicken pox is a common disease of childhood. It is caused by which

of the following viruses?

a. Cytomegalovirus

b. Rotavirus

c. Varicella-zoster virus

d. Adenovirus

e. Papillomavirus

83. Excluding influenza, which one of the following viruses is a common


cause of acute respiratory disease?

a. Cytomegalovirus

b. Rotavirus

c. Varicella-zoster virus

d. Adenovirus

e. Papillomavirus

84. Human warts are not only cosmetically unsightly but may lead to cancer

of the cervix. They are caused by which one of the following viruses?

a. Cytomegalovirus

b. Rotavirus

c. Varicella-zoster virus

d. Adenovirus

e. Papillomavirus

85. A vaccine is available for one of the most common causes of infantile

gastroenteritis. However, it has recently been recalled. The virus is

a. Cytomegalovirus

b. Rotavirus

c. Varicella-zoster virus

d. Adenovirus

e. Papillomavirus

86. A child has mononucleosis-like symptoms yet the test for mononucleosis

and the EBV titers are negative. One of the causes of heterophilenegative

mononucleosis is

a. Cytomegalovirus

b. Herpes simplex virus

c. Varicella-zoster virus

d. Adenovirus

e. Coxsackievirus

24 Microbiology
87. Malaise and fatigue with increased â atypicalâ lymphocytes and a reactive

heterophil antibody test is most commonly caused by

a. Toxoplasma

b. Borrelia burgdorferi

c. Epstein-Barr virus

d. Parvovirus

e. Rubella virus

88. Lethargy, malaise, and fatigue are observed in a patient 2 weeks after

eating raw hamburger at a restaurant. The most likely infectious cause is

a. Toxoplasma

b. Cytomegalovirus

c. E. coli

d. Salmonella

e. Clostridium

89. Burkittâ s lymphoma is characterized by elevated â early antigenâ tests

with a restricted pattern of fluorescence. This disease is caused by

a. Cytomegalovirus

b. B. burgdorferi

c. Epstein-Barr virus

d. Lymphogranuloma venereum

e. Herpes simplex virus

90. This virus may be detected by the polymerase chain reaction (PCR) in

a variety of cells of patients with nasopharyngeal carcinoma.

a. Measles

b. Mumps

c. Rubella

d. Parvovirus

e. Epstein-Barr virus

Virology 25
91. This virus causes a mononucleosis-like syndrome caused by a latent

herpesvirus; it is often a congenital infection. Large amounts of the virus

are excreted in the urine; thus, urine becomes the fluid of choice for diagnosis

of this disease.

a. Epstein-Barr virus

b. Cytomegalovirus

c. HHV-6

d. Parvovirus

e. Norwalk virus

Questions 92â 96

Assume you are asked by a resident what the most appropriate specimen is

for the detection of a particular virus.

92. Human papillomavirus

a. Cervical tissue

b. Synovial fluid

c. Blood

d. Skin

93. Cytomegalovirus

a. Cervical tissue

b. Synovial fluid

c. Blood

d. Skin

e. Cerebrospinal fluid

94. Enterovirus

a. Cervical tissue

b. Synovial fluid

c. Blood

d. Skin

e. Cerebrospinal fluid
26 Microbiology

95. Varicella-zoster virus (VZV)

a. Cervical tissue

b. Synovial fluid

c. Blood

d. Skin

e. Cerebrospinal fluid

96. Adenovirus 40/41

a. Cervical tissue

b. Synovial fluid

c. Blood

d. Stool

e. Cerebrospinal fluid

97. Which of the following is transmitted by the fecal-oral route; can be

acquired from shellfish; and often causes acute jaundice, diarrhea, and liver

function abnormalities?

a. Rotavirus

b. Adenovirus 40/41

c. Norwalk virus

d. Astrovirus

e. Hepatitis A virus

98. Which of the following is the second most common cause of pediatric

gastroenteritis? Unlike other similar viruses, this virus causes only gastroenteritis.

a. Rotavirus

b. Adenovirus 40/41

c. Norwalk virus

d. Astrovirus

e. Hepatitis A virus

99. Which of the following is the most common cause of pediatric gastroenteritis?
It is difficult to grow in cell culture but can be detected easily

by immunologic methods (ELISA).

a. Rotavirus

b. Adenovirus 40/41

c. Norwalk virus

d. Astrovirus

e. Hepatitis A virus

Virology 27

100. Which of the following is a common cause of epidemic gastroenteritis,

particularly aboard cruise ships and in summer camps? It may be

detected by ELISA methods or electron microscopy.

a. Rotavirus

b. Adenovirus 40/41

c. Norwalk virus

d. Astrovirus

e. Hepatitis A virus

101. Which of the following is a cause of mild gastroenteritis? It can be

transmitted by the fecal-oral route but not by food consumption.

a. Rotavirus

b. Adenovirus 40/41

c. Norwalk virus

d. Astrovirus

e. Hepatitis A virus

102. IgM antibody to the viral particle is the method of choice for laboratory

diagnosis of which one of the following hepatitis viruses?

a. Hepatitis A

b. Hepatitis B

c. Hepatitis C

d. Hepatitis D
e. Hepatitis E

103. This virus belongs to the family of flaviviruses and its reservoir is

strictly human. Transmission is blood-borne so the blood supply is routinely

screened for this virus.

a. Hepatitis A

b. Hepatitis B

c. Hepatitis C

d. Hepatitis D

e. Hepatitis E

28 Microbiology

104. Vaccination for this hepatic disease is with viral surface antigen and

usually provides immunity.

a. Hepatitis A

b. Hepatitis B

c. Hepatitis C

d. Hepatitis D

e. Hepatitis E

105. This hepatitis virus is a calicivirus. The reservoir is in pigs, and

humans acquire it via the fecal-oral route.

a. Hepatitis A

b. Hepatitis B

c. Hepatitis C

d. Hepatitis D

e. Hepatitis E

106. This hepatitis virus is a defective virus in that it cannot replicate

independently without the presence of hepatitis B virus.

a. Hepatitis A

b. Hepatitis B

c. Hepatitis C
d. Hepatitis D

e. Hepatitis E

107. Which of the following is the causative agent of a variety of cutaneous

warts (plantar, common, and flat) and is associated with cervical neoplasia?

a. Human papillomavirus

b. West Nile virus

c. Tick-borne encephalitis virus

d. Polyomavirus

e. Subacute sclerosing panencephalitis virus (SSPE)

108. Recently appearing in the United States, this virus is carried by birds,

transmitted by mosquitoes, and infects humans and horses.

a. Human papillomavirus

b. West Nile virus

c. Tick-borne encephalitis virus

d. Polyomavirus

e. SSPE

Virology 29

109. Which of the following viruses causes progressive multifocal

leukoencephalopathy (PML), a disease causing demyelination in the central

nervous system?

a. Human papillomavirus

b. West Nile virus

c. Tick-borne encephalitis virus

d. Polyomavirus

e. SSPE

110. This virus is transmitted by the same arthropod that transmits

babesiosis and ehrlichiosis.

a. Human papillomavirus

b. West Nile virus


c. Tick-borne encephalitis virus

d. Polyomavirus

e. SSPE

111. This virus is a single-stranded RNA orthomyxovirus. Annual vaccination

is necessary because of antigenic drift and shift.

a. Measles virus

b. Influenza virus

c. Respiratory syncytial virus

d. Parainfluenza virus

e. Adenovirus

112. This virus is a single-stranded RNA paramyxovirus. The rash known

as Koplikâ s spots is pathognomonic.

a. Measles virus

b. Influenza virus

c. Respiratory syncytial virus

d. Parainfluenza virus

e. Adenovirus

113. This virus is the leading cause of bronchiolitis and communityacquired

pneumonia in infants.

a. Measles virus

b. Influenza virus

c. Respiratory syncytial virus

d. Parainfluenza virus

e. Adenovirus

30 Microbiology

114. This is a paramyxovirus and causes the syndrome known as croup.

a. Measles virus

b. Influenza virus

c. Respiratory syncytial virus


d. Parainfluenza virus

e. Adenovirus

115. This is a double-stranded DNA virus. It is responsible for 15% of

pediatric respiratory infections and 10 to 15% of acute diarrhea in children.

a. Measles virus

b. Influenza virus

c. Respiratory syncytial virus

d. Parainfluenza virus

e. Adenovirus

You might also like